The Practice Question of the Week will help you stay informed of state-of-the art practices for the prevention, treatment, and care of people with or at risk of HIV, COVID-19, or other viral infections. The questions will test and reinforce your current practices or knowledge. You will receive immediate feedback on the correct response and rationale, as well as further IAS–USA resources on these key topics.

April 15, 2024: Long-Acting Injectable Cabotegravir (CAB-LA) for HIV Preexposure Prophylaxis

Previous Questions

Which of the following statements about long-acting injectable cabotegravir (CAB-LA) for HIV preexposure prophylaxis is TRUE?

Responses
A. CAB-LA is less effective in the setting of active bacterial sexually transmitted infections 43 (6%)
B. Every 2-month thigh injections of CAB-LA resulted in similar pharmacokinetics to every 2-month gluteal injections 241 (33%)
C. During CAB-LA follow-up, the combination of a rapid HIV test and antigen/antibody test or 2 rapid tests that give the same result (eg, both positive or both negative) had high positive and negative predictive values 306 (41%)
D. Integrase strand transfer inhibitor resistance mutations are unlikely to emerge in the setting of breakthrough infections with CAB-LA 148 (20%)

Correct answer is C. The high positive and negative predictive values of concordant rapid and antigen/antibody test results suggest this testing may be sufficient for follow-up long-acting cabotegravir (CAB-LA) visits.

To learn more about this topic, be sure to register for our upcoming webinar, presented by Susan P. Buchbinder, MD, and moderated by Albert Y. Liu, MD, MPH, The ART of HIV and STI Prevention: Update From the 2024 Conference on Retroviruses and Opportunistic Infections (CROI).

Which of the following is TRUE about the LATITUDE (Long-Acting Therapy to Improve Treatment Success in Daily Life) study (CROI Abstract 5359)?

Responses
A. It was not a randomized, clinical study 55 (7%)
B. All participants had virologic suppression at the time they started long-acting cabotegravir/rilpivirine (LA CAB/RPV) 250 (30%)
C. In the LA CAB/RPV group, on-time dosing only occurred 70% of the time 73 (9%)
D. Those given LA CAB/RPV had lower rates of virologic and treatment-related failure 448 (54%)

Correct answer is D. There was a lower rate of virologic and treatment-related failure in those randomized to LA CAB/RPV, which led to the premature termination of the LATITUDE study. A, B, and C are not true, respectively, because the study was a randomized, clinical trial, approximately 15% of the study participants had detectable virus at the time the study regimen was initiated, and the timing of LA CAB/RPV dosing was on time more than 90% of the time.

To learn more about this topic, register for webinar, Antiretroviral Management of HIV: New Data From the 2024 Conference on Retroviruses and Opportunistic Infections (CROI), to be presented by Eric S. Daar, MD, and moderated by Judith S. Currier, MD, on April 12, 2024.

For more information on this topic, be sure to listen to the latest episode of the Going anti-Viral podcast: 

Episode 14 – HIV Treatment with Long-Acting Injectables with Dr Aadia Rana Recorded Live at CROI 2024 on March 4, 2024
(https://www.iasusa.org/going-anti-viral-podcast/)

And read more here:

Sax PE, Thompson MA, Saag MS, IAS–USA Treatment Guidelines Panel. Updated treatment recommendation on use of cabotegravir and rilpivirine for people with HIV from the IAS–USA Guidelines Panel. JAMA. 2024;331(12):1060-1061. doi:10.1001/jama.2024.2985

Rana AI, Bao Y,  Zheng L, et al. Long-acting injectable CAB/RPV is superior to oral ART in PWH with adherence challenges: ACTG A5359 [CROI Abstract 212]. In Special Issue: Abstracts From the CROI 2024 Conference on Retroviruses and Opportunistic Infections. Top Antivir Med. 2024;32(1):57.

Which of the following statements is TRUE?

Responses
A. Once-weekly lenacapavir/bictegravir showed 96% viral suppression in persons without prior treatment 139 (37%)
B. Once-daily doravirine/islatravir 0.25 mg showed a 30 cells/µL decrease in CD4+ count at 48 weeks 61 (16%)
C. GS-1720 is an oral integrase strand transfer inhibitor with a half-life that supports weekly dosing 146 (38%)
D. The nonnucleoside reverse transcriptase inhibitor MK-8507 caused lymphopenia when given with islatravir 34 (9%)

Correct answer is C. GS-1720 is an oral integrase strand transfer inhibitor with a half-life that supports weekly dosing. Once-daily, not once-weekly, lenacapavir/bictegravir achieved high rates of viral suppression among study participants. Dosage for once-daily doravirine/ islatravir is 0.75 mg, not 0.25 mg. Lymphopenia was caused by islatravir, not MK-8507.

To learn more about this topic, watch the on-demand webinar, New Drugs on the Horizon for HIV Treatment: 2024 and Beyond!, presented by Melanie A. Thompson, MD, and moderated by Rajesh T. Gandhi, MD, on March 29, 2024.

Which statement accurately contrasts acute HIV infection (AHI) with long-acting early viral inhibition (LEVI) in the context of HIV?

Responses
A. AHI is characterized by explosive viral replication immediately following infection, whereas LEVI involves smoldering viral replication due to long-acting antiviral preexposure prophylaxis (PrEP) agents 633 (78%)
B. LEVI is typically detected through standard antigen/antibody assays, RNA assays, and DNA assays 79 (10%)
C. LEVI is associated with a high likelihood of HIV transmission 40 (5%)
D. Drug resistance is a common feature of AHI because of natural infection dynamics 59 (7%)

Correct answer is A. Long-acting early viral inhibition (LEVI) is an alteration of the traditional biology associated with HIV acquisition and has been seen with long-acting preexposure prophylaxis (PrEP) with cabotegravir. LEVI is characterized by smoldering viral replication (not explosive), minimal resistance (unless transmitted), and diagnostic challenges when using conventional HIV diagnostic assays such as rapid tests and antigen/antibody assays. Because of the low viral loads associated with LEVI, transmission outside of transfusion-related theoretical possibilities would be anticipated to be rare, in notable contrast to more classic acute or primary HIV infection.

To learn more about this topic, watch the on-demand webcast, Long-Acting Agents for HIV Therapeutics and Prevention: Where We Are, Where We Aren’t, and Where We’re Going, presented by Raphael J. Landovitz, MD, MSc, at the Scott M. Hammer Annual Update on HIV Management in New York, New York, on March 18, 2024.

Which of the following is a potential mechanism of weight suppression observed with tenofovir disoproxil fumarate (TDF) presented at this year’s CROI?

Responses
A. Endogenous stimulation of glucagon-like peptide-1 286 (42%)
B. Damage to intestinal enterocytes and alteration of structure 225 (33%)
C. Appetite suppression by a direct effect on the hypothalamus 98 (15%)
D. Premature satiety induced by slowed gastric emptying 71 (10%)

Correct answer is B. In various studies, the use of tenofovir disoproxil fumarate (TDF) has been associated with less weight gain than comparator regimens. In this study, 12 people on TDF-based treatment and 12 on tenofovir alafenamide fumarate (TAF)-based treatment underwent upper endoscopy, with biopsies of the duodenum. Those on TDF had more histologic abnormalities (flatter villi and deeper crypts), as well as lower levels of certain nutrients absorbed from the proximal duodenum, and higher levels of serum intestinal fatty acid-binding protein, a marker of enterocyte damage.

To learn more about this topic, watch the on-demand webcast, Key Updates from CROI 2024, presented by Paul E. Sax, MD, at the Scott M. Hammer Annual Update on HIV Management in New York, New York, on March 18, 2024.

Is it reasonable to begin a patient who was virally suppressed on tenofovir alafenamide fumarate/emtricitabine/bictegravir with a history of K103N on genotyping (only) on cabotegravir/rilpivirine?

Responses
A. Yes 313 (28%)
B. No 189 (17%)
C. Yes, if there has been no prior failure on a nonnucleoside reverse transcriptase inhibitor-based regimen 564 (51%)
D. Yes, if they are not taking a proton pump inhibitor 45 (4%)

Correct answer is C. Cabotegravir (CAB)/rilpivirine (RPV) is approved for the treatment of HIV infection absent any resistance to component agents and in the setting of current virologic suppression on oral agents. K103N should not, by itself, confer resistance to RPV, and therefore if otherwise eligible, K103N would not be a contraindication to the use of CAB/RPV. However, K103N may also be a marker for other nonnucleoside reverse transcriptase inhibitor (NNRTI) resistance, and any history of virologic failure on NNRTIs-or some would be as conservative as to say NNRTI exposure-may temper the excitement for using CAB/RPV. Use of a proton pump inhibitor, although a contraindication to the use of oral RPV, would not affect absorption/metabolism of injectable RPV, but in this case, the use of an oral lead-in would be contraindicated.

To learn more about this topic, watch the on-demand webcasts of the course, The Scott M. Hammer Annual Update on HIV Management in New York, New York, which will be available soon.

A patient who previously had mpox asks about their vaccination options. Which of the following is the current recommendation by the Advisory Committee on Immunization Practices (ACIP)?

 

Responses
A. 1 injection with the modified vaccinia Ankara (MVA) vaccine 130 (10%)
B. 2 injections, 4 weeks apart, with the MVA vaccine 436 (33%)
C. 1 injection with the ACAM2000 smallpox vaccine (Sanofi) 112 (8%)
D. No vaccination is recommended 648 (49%)

Correct answer is D. The Advisory Committee on Immunization Practices recommends no vaccination for those who have previously had mpox.

To learn more about this topic, watch the on-demand webinar, Mpox: Where Are We Now? presented by Jason E. Zucker, MD, and moderated by Judith S. Currier, MD, on February 29, 2024.

Which of the following antiviral treatments has been shown to be effective against mpox in humans in a randomized clinical trial (RCT)?

Responses
A. Tecovirimat 931 (43%)
B. Cidofovir 84 (4%)
C. Brincidofovir 53 (2%)
D. Acyclovir 98 (4%)
E. No antiviral treatment has been shown to be effective in an RCT 1018 (47%)
Correct answer is E. No antiviral treatment has been shown to be effective in a randomized controlled trial.
To learn more about this topic, watch the on-demand webinar, Mpox: Where Are We Now? presented by Jason E. Zucker, MD, and moderated by Judith S. Currier, MD, on February 29, 2024.

According to recently released guidance by the Centers for Disease Control and Prevention, doxycycline postexposure prophylaxis (doxyPEP) is recommended for which of the following populations?

Responses
 A. Gay, bisexual, and other men who have sex with men (MSM) with no prior bacterial sexually transmitted infections (STIs) in the last 12 months 34 (4%)
B. Gay, bisexual, and other MSM with at least 1 bacterial STI in the last 12 months 390 (51%)
C. All sexually active men and women with no prior bacterial STIs in the last 12 months 39 (5%)
D. All sexually active men and women with at least 1 bacterial STI in the last 12 months 121 (16%)
E. Any individual who feels that they are at risk for a bacterial STI, regardless of STI history 181 (24%)

Correct answer is B. The Centers for Disease Control and Prevention released draft guidance on the use of doxycycline postexposure prophylaxis (doxyPEP) in late September 2023. Their recommendation for the use of doxycycline for the prevention of bacterial sexually transmitted infections (STIs) is limited to gay, bisexual, and other men who have sex with men who have had at least 1 bacterial STI in the last 12 months. This recommendation is based on studies published in the last year documenting up to two-thirds reduction in bacterial STIs among gay/bisexual men. To date, there are no definitive data on women or men who have sex with women, but many experts feel that doxyPEP will work in these populations as well. In areas with high bacterial STI rates, some clinicians have adopted the use of doxyPEP in all persons who have had at least 1 STI in the last 12 months, while definitive data are awaited from ongoing studies.

For more information on this topic, be sure to listen to the latest episode of the Going anti-Viral podcast, Understanding the Implementation of Doxycycline Postexposure Prophylaxis (DoxyPEP) and Addressing Sexually Transmitted Infections, With Dr Annie Luetkemeyer https://www.iasusa.org/2024/02/13/going-anti-viral-podcast-episode-9/

References

  1. Centers for Disease Control and Prevention. Guidelines for the use of doxycycline post-exposure prophylaxis for bacterial sexually transmitted infection (STI) prevention; request for comment and informational presentation. October 2, 2023. Accessed February 19, 2024. https://www.federalregister.gov/documents/2023/10/02/2023-21725/guidelines-for-the-use-of-doxycycline-post-exposure-prophylaxis-for-bacterial-sexually-transmitted
  2. Molina JM, Charreau I, Chidiac C, et al. Post-exposure prophylaxis with doxycycline to prevent sexually transmitted infections in men who have sex with men: an open-label randomised substudy of the ANRS IPERGAY trial. Lancet Infect Dis. 2018;18(3):308-317. Epub 2017 Dec 8. doi:10.1016/S1473-3099(17)30725-9
  3. Luetkemeyer AF, Donnell D, Dombrowski JC, et al. Postexposure doxycycline to prevent bacterial sexually transmitted infections. N Engl J Med. 2023;388(14):1296-1306. doi:10.1056/NEJMoa2211934
  4. Molina JM, Bercot B, Assoumou L, et al. ANRS 174 DOXYVAC: an open-label randomized trial to prevent STIs in MSM on PrEP. [CROI Abstract 119]. In Special Issue: Abstracts From CROI 2023 Conference on Retroviruses and Opportunistic Infections. Top Antivir Med. 2023;31(2):49.
  5. Stewart J, Oware K, Donnell D, et al. Self-reported adherence to event-driven doxycycline postexposure prophylaxis for sexually transmitted infection prevention among cisgender women. [CROI Abstract 121]. In Special Issue: Abstracts From CROI 2023 Conference on Retroviruses and Opportunistic Infections. Top Antivir Med. 2023;31(2):49-50.

You are treating a 28-year-old man who was recently hospitalized for osteomyelitis resulting from injection drug use (IDU) of opioids and stimulants. He has posttraumatic stress disorder, opioid use disorder, stimulant use disorder, and HIV. He lives in a rural area with limited access to substance use disorder (SUD) treatment. He is doing well on antiretroviral therapy through your clinic but is still struggling with IDU. He has done well on sublingual buprenorphine (SL-bup) in the past but struggled with weekly appointments and filling weekly scripts at the pharmacy because of transportation problems. What do you recommend for his SUD treatment?

Responses
  93 (21%)
B. Recommend long-acting injectable buprenorphine (LAI-bup) because he liked SL-bup and LAI-bup may be less of a transportation burden 301 (67%)
C. Recommend injectable naltrexone because buprenorphine treatment failed, and this will be less of a transportation burden 31 (7%)
D. Recommend reinitiating SL-bup because he liked it, and if he likes it, he should be able to arrange transportation with the case manager 22 (5%)

Correct answer is B. Buprenorphine treatment has not failed because the patient had transportation problems. Methadone treatment often requires months of daily visits to the clinic and would be more challenging for the patient. You could discuss and offer naltrexone, but given that he liked sublingual buprenorphine, it is reasonable to offer long-acting injectable buprenorphine to see if he would like to try that. It would also be helpful to ask the patient why he liked buprenorphine (eg, did he find that it relieved his withdrawal, diminished cravings or urges to use opioids or reduced illicit opioid use). It is important to provide hope, set the patient up for success, and involve the patient in the decision-making process.

For more information on this topic, view the on-demand webinar, (Part 6 of 6) Substance Use Disorder: Future Directions Focusing on Long-Acting Injectable Buprenorphine, presented by Michelle R. Lofwall, MD, and moderated by Ellen F. Eaton, MD.

The 6-part IAS–USA webinar series on substance use disorder (SUD), designed by expert faculty Sandra A. Springer, MD, and Ellen F. Eaton, MD, has been a resounding success, with 6 webinars comprising 8 hours of DEA-compliant content, plus a bonus Q and A session, now available free and on-demand through 2026! To learn more, visit our MATE Act CME page at https://www.iasusa.org/dea-compliant-cme-resource-center/

Which of the following statements is TRUE about lenacapavir?

Responses
A. Lenacapavir resistance is common in wild type viruses 22 (3%)
B. The M66I capsid mutation was the most frequently observed mutation in highly treatment-experienced patients in whom lenacapavir failed 471 (62%)
C. No emerging resistance was observed in highly treatment-experienced patients in whom lenacapavir failed 124 (16%)
D. No emerging resistance was observed in drug-naive patients in whom lenacapavir failed 85 (11%)
E. Genotypic resistance testing for lenacapavir is not important in clinical practice 62 (8%)

Correct answer is B. The M66I capsid mutation was the most frequently observed mutation in highly treatment-experienced patients in whom lenacapavir failed.

To learn more about the principles of HIV drug resistance mutations and what is currently known about resistance to antiretroviral drugs and products, including new and long-acting medications, watch the on-demand webinar, Basic Principles and Clinical Relevance of the IAS–USA 2022 Update of the Drug Resistance Mutations in HIV-1, presented by Francesca Ceccherini Silberstein, PhD, Annemarie M. Wensing, MD, PhD, and moderated by Douglas D. Richman, MD, on January 31, 2024.

In which demographic group has the rate of syphilis increased dramatically in recent years?

Responses
A. Adult heterosexual women  411 (43%)
B. Adult heterosexual men 42 (4%)
C. Adult gay men, bisexual men, and other men who have sex with men 429 (44%)
D. Adolescents 64 (7%)
E. None of the above 21 (2%)

Correct answer is A. There has been a substantial increase in syphilis rates among heterosexual women in recent years. From 2020 to 2021, the rate of primary and secondary syphilis among women increased by more than 50%. This increase is associated with a corresponding escalation in the rate of congenital syphilis, which has increased each year since 2013.

To learn more about this topic, subscribe now to the IAS–USA podcast, Going anti-Viral, hosted by Dr Michael S. Saag, and listen to, Episode 5 – An In-depth Discussion on Syphilis with Dr Meredith Clement, Associate Professor at Louisiana State University Health Sciences Center. Don’t miss out on upcoming episodes, where Dr Saag interviews an expert in infectious diseases or emerging pandemics about their area of specialty and current developments in the field – subscribe now!

Which of the following is true about long-acting injectable buprenorphine (LAI-bup) formulations?

 

Responses
A. LAI-bup comes in different doses and can be given once a week, once a month, or sometimes less than once a month.  457 (56%)
B. LAI-bup is contraindicated with preexposure prophylaxis and long-acting antiretroviral therapy. 55 (7%)
C. LAI-bup should not be given to pregnant and breastfeeding persons. 213 (26%)
D. The patient goes to the pharmacy to pick up LAI-bup, where the pharmacist teaches them how to self-inject the dose. 86 (11%)

Correct answer is A. There are weekly and monthly doses of long-acting injectable buprenorphine (LAI-bup), and the 300 mg dose of RBP-6000 can be administered to patients on 100 mg monthly maintenance doses if they are going to be away for more than a month and would miss their scheduled 100 mg dose. Although there are scant data on the use of LAI-bup with preexposure prophylaxis and long-acting injectable HIV medications, there is no contraindication to giving both to the same patient. Patients receiving LAI-bup should never have this medication in their possession. The medication should be directly administered to the patient by the clinician.

For more information on this topic, view the on-demand webinar, (Part 6 of 6) Substance Use Disorder:

Future Directions Focusing on Long-Acting Injectable Buprenorphine, presented by Michelle R. Lofwall, MD, and moderated by Ellen F. Eaton, MD.

What is the most effective public health intervention to prevent the development of long COVID, according to current evidence?

Responses
A. Masks  1572 (18%)
B. SARS-CoV-2 vaccination 2210 (26%)
C. Early antiviral treatment 1605 (19%)
D. Corticosteroid therapy 1537 (18%)
E. Hand washing 1597 (19%)

Correct answer is B. Current evidence shows that the risk of developing long COVID is consistently 30% to 60% lower in people who are correctly vaccinated with at least 2 to 3 doses of SARS-CoV-2 vaccine.

To learn more about this topic, watch the on-demand webinar, Pathogenesis of Long COVID, presented by Roger Paredes, MD, PhD, and moderated by Michael J. Peluso, MD, on January 16, 2024.

Which of the statements following is true regarding the pathogenesis of long COVID?

 

Responses
A. The pathogenesis of long COVID is unknown.  54 (7%)
B. Long COVID is a psychosomatic disease. 18 (2%)
C. The pathogenesis of long COVID is not fully understood, but there is strong evidence of early, diverse, and persistent organ and tissue damage. 493 (60%)
D. Although not fully understood, there are a number of long COVID biomarkers that have been clinically and technically validated and have been useful in determining a clinical definition of long COVID. 212 (26%)
E. Long COVID pathogenesis is like that of other postviral diseases. 42 (5%)

Correct answer is C. There is solid and cumulative evidence of early and persistent organ and tissular damage in subjects with long COVID, which is also supported by animal models and in vitro studies. Most patients do not have altered mental test results when they are diagnosed with long COVID, although anxiety, depression, and other mental health concerns may develop or aggravate during the course of the disease, particularly when patients realize that their symptoms will not remit easily. Considering long COVID a “mental disease” is stigmatizing for patients with long COVID and with mental disorders. Although plasma antigenemia is promising, it still requires further technical and clinical validation. Finally, viruses are different in many ways (antigenically, mechanistically, in terms of tropism, etc), so the different postviral syndromes do not necessarily share the same intimate pathogenic mechanisms.

To learn more about this topic, watch the on-demand webinar, Pathogenesis of Long COVID, presented by Roger Paredes, MD, PhD, and moderated by Michael J. Peluso, MD, on January 16, 2024.

Which of the following is a US Food and Drug Administration (FDA)-approved medication for addiction?

Responses
A. Topiramate  42 (8%)
B. Mirtazapine42 25 (5%)
C. Extended-release naltrexone
224 (45%)
D. Extended-release naltrexone/bupropion combination 207 (42%)

Correct answer is C. Extended-release naltrexone is approved for the treatment of opioid use disorder and alcohol use disorder.

Although topiramate is supported by the American Society of Addiction Medicine (ASAM) practice guidelines for treating cocaine use disorder, especially when combined with extended-release mixed amphetamine salts, this medication is not US Food and Drug Administration (FDA)-approved for an addiction indication.

Similarly, although mirtazapine and extended-release naltrexone/bupropion combination have strong evidence for use to treat methamphetamine use disorder, none of these drugs are FDA-approved for an addiction indication.

To learn more about stimulant use disorder in patients with HIV, view the on-demand webinar: (Part 5 of 6) Substance Use Disorder: Stimulant Use Disorder, Harm Reduction, and Contingency , presented by Steven J. Shoptaw, PhD, and moderated by Sandra A. Springer, MD.

A 53-year-old obese woman presents to care with HIV infection. She has been doing well on antiretroviral treatment, and her last plasma HIV RNA level 6 months ago was below the limits of detection. She takes medications for schizophrenia and type 2 diabetes mellitus. Overall, the patient is on 7 different prescribed medications for her 3 conditions. She presents today with a sudden increase in auditory hallucinations. Her vital signs are unchanged from their usual baseline. Which of the following is the most appropriate next step?

Responses
A. Assess the patient for problems caused by drug interactions 388 (72%)
B. Refer the patient for an increase in her antipsychotic medication 18 (3%)
C. Check the patient’s blood sugar 123 (23%)
D. Send the patient for brain imaging 13 (2%)

Correct answer is C. This patient was initially referred for an increase in her antipsychotic medication. The evaluating psychiatrist first ordered laboratory tests which revealed a blood glucose level of 600 mg/dL. Treatment of the patient’s elevated blood sugar restored the patient to her baseline mental status. Always first look for an urgent medical problem when assessing a change in mental status.

 

To learn more about how addressing mental health directly reduces morbidity and mortality in people with HIV, view this webcast from 2023 Ryan White CLINICAL CONFERENCE, Beyond Viral Suppression: How Addressing Mental Health Directly Reduces Morbidity and Mortality in People with HIV, presented by Francine Cournos, MD. You may also earn continuing education credits through the on-demand activity here.

Which of the following has the highest signal strength for treating stimulant use disorder?

Responses
A. Extended-release naltrexone/bupropion combination 332 (40%)
B. Refer the patient for an increase in her antipsychotic medication 48 (6%)
C. Check the patient’s blood sugar 193 (23%)
D. Send the patient for brain imaging 260 (31%)

Correct answer is D. Contingency management has numerous meta-analyses showing the signal strength of the treatment compared with the standard-of-care is in the moderate-to-strong range. As such, this treatment has the highest signal of efficacy compared with all other treatments, behavioral or pharmacological. The largest well-controlled, multisite trial of any medication for methamphetamine use disorder in the broad population with efficacy for reducing methamphetamine use is the combination of extended-release naltrexone plus bupropion, but the strength of signal for the combination medication is much smaller than that of contingency management. Although mirtazapine has 2 replication studies, which is a marker of the consistency of the signal for the medication to reduce methamphetamine use compared with placebo, to date the medication has only been evaluated in clinical samples of men who have sex with men and transgender women. There are no placebo-controlled studies of mirtazapine in broad groups of people with methamphetamine use disorder. Moreover, the size of signal for mirtazapine is far less than that for contingency management. Although cognitive behavior therapy is used widely in addiction care settings, the signal strength for it is far lower than for contingency management.

 

To learn more about contingency management and stimulant use disorder, register now for our December 20, 2023, webinar, (Part 5 of 6) Substance Use Disorder: Stimulant Use Disorder, Harm Reduction, and Contingency Management in Patients With HIV, presented by Steven J. Shoptaw, PhD, and moderated by Sandra A. Springer, MD.

Glucagon-like peptide-1 receptor agonist (GLP-1RAs) have shown several potential off-target effects. Which of the following has been demonstrated in randomized control trials?

Responses
A. Reducing visceral fat, increasing subcutaneous fat, increasing lean body mass 31 (4%)
B. Reducing visceral fat, reducing subcutaneous fat, increasing lean body mass 303 (38%)
C. Reducing visceral fat, reducing subcutaneous fat, reducing lean body mass 381 (48%)
D. Reducing visceral fat, increasing subcutaneous fat, reducing lean body mass 78 (10%)

Correct answer is C. The STEP 1 (Effect and Safety of Semaglutide 2.4 mg Once-weekly in Subjects With Overweight or Obesity) trial evaluating semaglutide subcutaneous weekly versus A placebo over 68 weeks demonstrated that among persons with obesity, the percentage change in body weight was -15.0% with semaglutide versus -3.6% with placebo. The decrease in body weight resulted from reductions in total fat mass, total lean body mass, and visceral fat mass. Additionally, recent studies evaluating semaglutide subcutaneous weekly versus a placebo over 32 weeks among persons with HIV-associated lipodystrophy with predominant lipohypertrophy similarly showed a decrease in total body weight with semaglutide use. Among those persons with HIV-associated lipohypertrophy treated with semaglutide, there was a decrease in visceral fat, subcutaneous fat, and total lean mass from baseline to 32 weeks. Although glucagon-like peptide-1 receptor agonists (GLP1-RAs) are useful to reduce total body weight, it is crucial to understand how different components of body composition may be affected by GLP-1RAs use. In this regard, preservation of subcutaneous fat or lean body mass is ideal among some clinical phenotypes, such as HIV-associated lipodystrophy with predominant lipoatrophy and sarcopenia, respectively.

References: Wilding JPH, Batterham RL, Calanna S, et al. Once-weekly semaglutide in adults with overweight or obesity. N Engl J Med. 2021;384(11):989-1002. doi:10.1056/NEJMoa2032183; McComsey GA, Sattar A, Wu Q, et al. Effects of semaglutide on adipose tissue in HIV-associated lipohypertrophy. Presented at: ID Week; October 13, 2023; Boston, MA.

 

To learn more about this topic, register for upcoming virtual course, The Annual IAS–USA Washington, DC, Virtual Course: Update on HIV Medicine Emerging Challenges, for Dr Suman Srinivasa’s presentation, “Weighing In” on GLP-1 Receptor Agonists for People With HIV.

Glucagon-like peptide-1 receptor agonist (GLP-1RAs) have shown several potential off-target effects. Which of the following has been demonstrated in randomized control trials?

Responses
A. GLP-1RAs reduce adverse cardiovascular outcomes in obesity among those with known atherosclerotic cardiovascular disease (ASCVD) 195 (28%)
B. GLP-1RAs reduce adverse cardiovascular outcomes in obesity among those with and without known ASCVD 146 (21%)
C. GLP-1RAs decrease heart failure events among those with heart failure with preserved ejection fraction and with obesity 71 (10%)
D. GLP-1RAs reduce the severity of substance use disorders 31 (4%)
E. All of the above 257 (37%)

Correct answer is A. The SELECT (Semaglutide Effects on Heart Disease and Stroke in Patients With Overweight or Obesity) trial evaluated cardiovascular outcomes among those with obesity without diabetes mellitus and demonstrated a 20% risk reduction with semaglutide subcutaneous weekly versus a placebo. The SELECT trial enrolled only those with pre-existing cardiovascular disease (CVD). The benefit to those without known CVD remains unclear at this time. (Reference: Lincoff AM, Brown-Frandsen K, Colhoun HM, et al. Semaglutide and cardiovascular outcomes in obesity without diabetes. N Engl J Med. Epub November 11, 2023. doi:10.1056/NEJMoa2307563)

In the Step-HFpEF (Research Study to Investigate How Well Semaglutide Works in People Living With Heart Failure and Obesity) trial, treatment with semaglutide among those with heart failure with preserved ejection fraction and obesity resulted in a large reduction in symptoms and improvements in exercise function. The study was not powered to assess heart failure events and further studies are needed. (Reference: Kosiborod MN, Abildstrøm SZ, Borlaug BA, et al. Semaglutide in patients with heart failure with preserved ejection fraction and obesity. N Engl J Med. 2023;389(12):1069-1084. doi:10.1056/NEJMoa2306963)

Preclinical studies have investigated GLP1-RAs to reduce addiction disorders and shown preliminary benefits to decrease substance use and drug-seeking behavior related to alcohol, nicotine, opioids, and simulants. Human studies have not thoroughly investigated GLP-1RA use for addiction disorders to date. (Reference: Jerlhag E. The therapeutic potential of glucagon-like peptide-1 for persons with addictions based on findings from preclinical and clinical studies. Front Pharmacol. 2023;14:1063033. Published 2023 Mar 30. doi:10.3389/fphar.2023.1063033)

To learn more about this topic, register for upcoming virtual course, The Annual IAS–USA Washington, DC, Virtual Course: Update on HIV Medicine Emerging Challenges, for Dr Suman Srinivasa’s presentation, “Weighing In” on GLP-1 Receptor Agonists for People With HIV.

A 35-year-old White man has been on efavirenz (EFV)/emtricitabine (FTC)/tenofovir disoproxil fumarate (TDF) for the past 10 years. He has been very reluctant to change a regimen that “saved his life.” However, because of persistent insomnia and depressive disorder, he is willing to consider a change. His CD4+ count is 700 cells/µL and his HIV RNA level is less than 20 copies/mL. Which of the following weight changes might result from a switch to bictegravir (BIC)/FTC/tenofovir alafenamide fumarate (TAF)?

Responses
A. No change in weight, as the patient was already virologically suppressed 65 (9%)
B. Weight loss, given that TAF is associated with fewer metabolic complications 28 (4%)
C. Weight gain because of the switch from TDF to TAF 86 (11%)
D. Weight gain because of the switch from EFV to BIC 74 (10%)
E. Both C and D 494 (66%)

Correct answer is E. Switching from tenofovir disoproxil fumarate (TDF) to tenofovir alafenamide fumarate (TAF) has been associated with a risk for increased weight, and so has a switch from efavirenz (EFV) to integrase strand transfer inhibitors. In a pooled analysis of randomized clinical trials, the greatest risk for weight gain was associated with the switch from EFV to rilpivirine (RPV) or elvitegravir (EVG)/cobicistat (COBI) and the switch from TDF to TAF. Switching from abacavir (ABC) to TAF was associated with less weight gain than the switch from TDF to TAF.

To learn more about this topic, see the on-demand webinar, HIV 101: Initial Assessment, Initiation of Antiretroviral Therapy, and Monitoring of People With Newly Diagnosed HIV, presented by Michael S. Saag, MD, Melanie A. Thompson, MD, and Roger J. Bedimo, MD, MS.

A 27-year-old cisgender woman was newly diagnosed with HIV this past week during a visit to a community clinic for a medroxyprogesterone acetate injection. Her HIV RNA level is 134,560 copies/mL, her CD4+ count is 662 cells/µL, and she is hepatitis B immune. Her baseline laboratory values are within normal limits, and genotypic resistance testing is pending. She is interested in starting therapy today and is asking for an injectable HIV treatment similar to her contraception. Based on the results of phase III trials and current HIV treatment guidelines, which of the following is true about initiating injectable antiretroviral therapy?

Responses
A. She can start injectable cabotegravir (CAB)/rilpivirine (RPV) today since we know she is hepatitis B immune 101 (21%)
B. She must first start oral CAB/RPV pills daily for 1 month before starting injectable CAB/RPV 34 (7%)
C. She will need to be on a stable oral antiretroviral therapy regimen for 3 to 6 months before transitioning to injectable CAB/RPV 276 (58%)
D. When she does start injectable CAB/RPV, it must be administered monthly for 6 months and then transition to injections every other month 14 (3%)
E. More information is needed 53 (11%)

Correct answer is C. Current US Department of Health and Human Services HIV treatment guidelines (November 2023) indicate long-acting cabotegravir (CAB) and rilpivirine (RPV) can be used to replace an existing oral antiretroviral regimen in people with HIV with sustained viral suppression for 3 to 6 months (AI). Once started, the injections can be administered either monthly or every other month based on a shared decision-making process. The oral CAB/RPV lead-in is optional as of March 2022.

To learn more about this topic, see the on-demand virtual course, Implementation of Long-Acting Drugs for Treatment and Prevention of HIV, for Dr Aadia I. Rana’s presentation, State of the Science: What We Know about Long-Acting Injectable Agents for Treatment of HIV.

At steady state after initial HIV infection, how many virions are produced in a 24-hour period?

Responses
A. 100 to 1000 virions/dayA. 100 to 1000 virions/dayA. 100 to 1000 virions/day 83 (7%)
B. 10,000 to 100,000 virions/day 231 (18%)
C. 1,000,000 to 10,000,000 virions/day 218 (17%)
D. 1,000,000,000 to 10,000,000,000 virions/day 419 (33%)
E. It depends on the viral load 320 (25%)

Correct answer is D. Studies of viral dynamics in the mid-1990s evaluated the slope of decay of viral load when patients first received highly active antiretroviral therapy. The slope demonstrated a very rapid decline in viral load due to the elimination of CD4+ lymphocytes that were actively producing HIV virus and not being replaced by newly infected cells. These findings demonstrated that the life span of a productively infected cell was approximately 1 day and that the number of viral particles produced in a day was between 1 billion and 10 billion particles per day. Although the number of viral particles produced in a 24-hour period may be a bit less in a patient with a lower viral load, it is still greater than 1 billion virions/day, but toward the lower end of the spectrum.

To learn more about this topic, view the webinar, HIV 101: Initial Assessment, Initiation of Antiretroviral Therapy, and Monitoring of People With Newly Diagnosed HIV, presented by Michael S. Saag, MD, Roger J. Bedimo, MD, MS, and Melanie A. Thompson, MD.

A 35-year-old man was diagnosed with HIV 3 years ago with an HIV RNA level of 57,000 copies/mL and a CD4+ count of 550 cells/µL. He started on bictegravir/emtricitabine/tenofovir alafenamide, suppressed rapidly, and has consistently had HIV RNA levels of less than 20 copies/mL (below the limit of detection) and CD4+ counts of greater than 750 cells/µL. He is hepatitis B virus immune, has a body mass index of 28, and has no comorbidities. He travels extensively for work and is interested in long-acting cabotegravir/rilpivirine administered every 2 months. Based on the results of phase III trials, which of the following is true about his risk of resistance if he follows the injection schedule and has on-time injections?

Responses
A. Confirmed virologic failure with resistance is very rare 612 (42%)
B. Confirmed virologic failure can happen but his risk is likely less than 1% over a 2- to 3-year period 581 (40%)
C. His risk of confirmed virologic failure is between 1% and 5% 203 (14%)
D. His risk of confirmed virologic failure is greater than 5% 13 (1%)
E. His risk of confirmed virologic failure is unknown 47 (3%)

Correct answer is B. Without any of the defined factors that increase the risk of confirmed virologic failure, his risk of confirmed virologic failure is approximately 0.4% over a 2- to 3-year period. The risk of resistance with confirmed virologic failure is approximately 80% to 90%.

To learn more about this topic, join us in the virtual course, Implementation of Long-Acting Drugs for Treatment and Prevention of HIV, for Dr Joseph J. Eron, Jr’s, presentation, Resistance in Long-Acting Therapy for Prevention and Treatment: Uncommon Outcome or Achilles Heel, which will be presented on November 13, 2023.

What is the main cause of cardiovascular disease (CVD) in people with HIV (PWH)?

Responses
A. CVD in PWH is due in large part to antiretroviral therapies, which can lead to metabolic abnormalities and account for the bulk of excess risk in PWH 78 (11%)
B. CVD in PWH is caused by the increased age of PWH, a population in which one would expect increased CVD 23 (3%)
C. CVD in PWH is caused by increased low-density lipoprotein, making it an ideal target for statin therapy 31 (4%)
D. CVD in PWH is caused by a combination of traditional and nontraditional risk factors, including inflammatory and immune-mediated mechanisms 570 (82%)

Correct answer is D. Traditional and nontraditional inflammatory risks contribute to cardiovascular disease (CVD) in people with HIV (PWH). The effects of antiretroviral therapies on metabolic disorders are thought to contribute only minimally to CVD in PWH. Although the population of PWH is aging, CVD can and often does occur in young PWH, who have premature CVD for their age. Low-density lipoprotein (LDL) is not often elevated, and statins were considered for both LDL and non-LDL effects.

To learn more about this topic, watch the on-demand webinar, Cardiovascular Disease Prevention in Patients With HIV: The REPRIEVE Trial, presented by Steven K. Grinspoon, MD, and moderated by Judith S. Currier, MD, on October 27, 2023.

Do data from REPRIEVE (Randomized Trial to Prevent Vascular Events in HIV) suggest the efficacy and overall utility of statins to prevent cardiovascular disease in low to moderate traditional-risk people with HIV who are in the 40- to 75-year age range?

 

Responses
A. Yes, based on efficacy and lack of unanticipated adverse effects in the REPRIEVE trial 690 (87%)
B. No, efficacy was shown to be marginal in the REPRIEVE trial 43 (6%)
C. No, because significant interactions with antiretroviral therapy were shown in the REPRIEVE trial results 33 (4%)
D. No, because the efficacy for major adverse cardiovascular event reduction was not seen among people developing diabetes mellitus in the trial 23 (3%)

Correct answer is A. Data from the global REPRIEVE (Randomized Trial to Prevent Vascular Events in HIV) trial suggest a 35% reduction in major adverse cardiac events, without unanticipated adverse effects, for a statin strategy among people with HIV who are aged 40 to 75 years with low to moderate risk by the pooled American College of Cardiology/American Heart Association equation. The efficacy effect was robust at 35%, including in sensitivity analyses. No significant interactions were seen with antiretroviral therapy. Among the small group with diabetes who suffered from a major adverse cardiac event, the rate was half in those on pitavastatin versus those on placebo. Protection was even conferred upon those with diabetes mellitus.

To learn more about this topic, watch the on-demand webinar, Cardiovascular Disease Prevention in Patients With HIV: The REPRIEVE Trial, presented by Steven K. Grinspoon, MD, and moderated by Judith S. Currier, MD on October 27, 2023.

Doxycycline taken after condomless sex has been shown to reduce incident bacterial sexually transmitted infections (eg, gonorrhea, chlamydia, and syphilis) by which percentage?

Responses
A. 95% 119 (14%)
B. 80% 299 (34%)
C. 65% 380 (44%)
D. 40% 61 (7%)
E. 10% 10 (1%)

Correct answer is C. Doxycycline postexposure prophylaxis (doxy-PEP) reduced the quarterly incidence of bacterial sexually transmitted infections by 65%. The reduction of chlamydia and syphilis incidence specifically was higher (approximately 80%) than the reduction of gonorrhea incidence (approximately 55%). To learn more about the efficacy of doxy-PEP and to earn up to 1.25 AMA PRA Category 1 Credits™, watch the On-Demand webinar, Doxy-Pep: Who, When, and How?, presented by Annie F. Luetkemeyer, MD, and moderated by Connie L. Celum, MD, MPH.

A 35-year-old woman with HIV and opioid use disorder is being admitted to the hospital for infectious complications of injection drug use. She will require intravenous antibiotics and will be in the hospital for 2 weeks. She has previously had a 3-year period of sobriety while on extended-release buprenorphine and would like to restart this treatment, but she has had repeated bouts of precipitated withdrawal when trying to initiate buprenorphine from fentanyl. She waits as long as she feels she can tolerate, but she still feels worse after taking 2 mg to 4 mg of buprenorphine. This has led to repeated fentanyl use. Which of the following is an appropriate strategy for medication for opioid use disorder initiation in this patient?

Responses
A. Begin a full-agonist opioid (eg, slow-release morphine) at a dosage sufficient to suppress withdrawal and then initiate very low-dose buprenorphine while continuing the full agonist. Titrate buprenorphine up slowly over several days and then stop full agonist. 321 (76%)
B. Methadone is her only option since she cannot tolerate withdrawal 28 (7%)
C. Offer her intramuscular extended-release naltrexone 33 (8%)
D. Stop all full-agonist opioids and begin very low-dose buprenorphine 40 (9%)

Correct answer is A. Protocols for low-dose buprenorphine initiation generally include a full-agonist opioid being given at a steady dosage for several days while buprenorphine is slowly increased to therapeutic levels. In the inpatient setting, this is done by replacing the street drug with an ordered full-agonist opioid (eg, slow-release morphine or oxycodone).

To learn more about medication options for opioid use disorder in patients with HIV, be sure to watch the on-demand webinar for (Part 3 of 6) Substance Use Disorder: Initiating Buprenorphine in the Fentanyl Era and Other Treatment Considerations in Patients With Opioid Use Disorder and HIV, presented by Leah J. Leisch, MD, and moderated by Sandra A. Springer, MD.

A 27-year-old woman with a past medical history of HIV presents with a request for help with opioid use disorder. She has been off antiretroviral therapy for the past 3 months but previously had plasma HIV RNA level suppression and no adverse events on lamivudine-zidovudine/efavirenz, and she would like to restart this regimen. She has tried quitting opioids several times, but efforts have been limited because of her inability to tolerate withdrawal symptoms. Her urine drug screen is positive for opiates and negative for fentanyl. Her last drug use was heroin the previous evening. The patient currently has stable housing but relies on friends for transportation. She has moderate withdrawal symptoms and feels very uncomfortable. Which medication for opioid use disorder would be the most appropriate to offer this patient in the clinic today?

Responses
A. Methadone 144 (19%)
B. Buprenorphine 497 (67%)
C. Naltrexone 66 (9%)
D. Naloxone 38 (5%)

Correct answer is B. It is most appropriate to offer this patient buprenorphine.

To learn more about medication options for opioid use disorder in patients with HIV, be sure to register for (Part 3 of 6) Substance Use Disorder: Initiating Buprenorphine in the Fentanyl Era and Other Treatment Considerations in Patients With Opioid Use Disorder and HIV, presented by Leah J. Leisch, MD, and moderated by Sandra A. Springer, MD.

Which of the following should be given as guidance when prescribing doxycycline postexposure prophylaxis (doxy-PEP)?

Responses
A. Doxycycline should not be given with ritonavir-boosted HIV protease inhibitors 107 (9%)
B. Doxycycline may protect against mpox 53 (5%)
C. Doxycycline should not be given with tenofovir-containing HIV preexposure prophylaxis 31 (3%)
D. Doxycycline should not be taken at the same time as bivalent-containing antacids, multivitamins, or calcium supplements 783 (67%)
E. Doxycycline should not be taken with acid-blocking medications like proton-pump inhibitors and histamine-2 blockers 187 (16%)

Correct answer is D. Doxycycline should not be taken at the same time as bivalent-containing antacids, multivitamins, or calcium supplements because of a drug interaction that may lower doxycycline levels. Doxycycline can be taken with HIV antiretroviral therapy, HIV preexposure prophylaxis, and acid-blocking medications. Doxycycline does not protect against viral infections, including mpox.

To learn more, watch the on-demand webinar, Doxy-PEP webinar, Doxy-PEP: Who, When, and How?, presented by Annie F. Luetkemeyer, MD, and moderated by Connie L. Celum, MD, MPH.

A 34-year-old man presents to care after a recent hospital admission for acute alcohol intoxication, complicated by a fall and pneumonia. He is diagnosed with alcohol use disorder. He has an 8-year history of HIV, with an HIV RNA level of 3000 copies/mL and he reports difficulty adhering to his antiretroviral therapy. Which of the following is the most appropriate pharmacotherapy for his alcohol use disorder?

Responses
A. Acamprosate, taken by mouth 3 times daily 84 (9%)
B. Disulfiram, taken by mouth daily 91 (9%)
C. Extended-release injectable naltrexone, administered every 4 weeks 664 (68%)
D. None of the above 138 (14%)

Correct answer is C. Because of difficulties with adherence to antiretroviral therapy (ART) regimens, it may be challenging for this patient to add another daily medication to treat alcohol use disorder (AUD). However, untreated AUD will impair his ability to adhere to ART and achieve viral-load suppression. Hence, treatment of AUD is essential to improving HIV outcomes. Extended-release naltrexone is a long-acting injectable treatment that can be delivered every 4 weeks and effectively improves viral suppression in persons with HIV and AUD. Oral naltrexone given daily at 25-50 mg can also reduce alcohol use and is approved by the US Food and Drug Administration for the treatment of AUD and hazardous drinking but adherence can be an issue, and it has more adverse effects than extended-release naltrexone. Disulfiram would require more monitoring as it can cause fatal ventricular fibrillation if someone continues to drink while on it. Acamprosate is also a potential option but requires oral dosing 3 times daily, which is a challenge for many patients

To learn more, watch the on-demand webinar: Substance Use Disorder (SUD) 101, Part 2: How Should We Care For SUD in Patients With HIV? presented by Ellen F. Eaton, MD, and moderated by Sandra A. Springer, MD.

A 30-year-old woman presents to care with HIV diagnosed 5 years ago. She is asymptomatic with active injection drug use. Her initial laboratory values included an HIV RNA level of 128,000 copies/mL and a CD4+ count of 150 cells/µL. She started on tenofovir disoproxil fumarate (TDF) / emtricitabine (FTC) / rilpivirine (RPV), with intermittent adherence. She now has an HIV RNA level of 12,000 copies/mL and a CD4+ count of 250 cells/µL. Genotypic testing detects M184V and K65R mutations. What regimen is LEAST likely to work?

Responses
A. TDF / lamivudine (3TC) / dolutegravir (DTG) 160 (24%)
B. TDF / 3TC / ritonavir-boosted darunavir (DRV/r) 126 (19%)
C. Zidovudine (ZDV) / 3TC / DTG 294 (45%)
D. DTG / DRV/r 76 (12%)

Correct answer is C. Based on the results of the NADIA (Nucleosides And Darunavir/Dolutegravir In Africa) and D²EFT (Dolutegravir and Darunavir Evaluation in Adults Failing Therapy) studies, ZDV-based regimens were the least successful; TDF + FTC paired with either DRV/r or DTG was statistically superior to the ZDV-based regimen. A combination of DTG + DRV/r was noninferior to DRV/r + 2 nucleoside reverse transcriptase inhibitors. The likely reason for poorer outcomes in ZDV-based regimens is intolerance to ZDV.

To learn more about antiretroviral therapy (ART) resistance, watch the on-demand webinar, What to Do Next: Use of Resistance Testing and Managing ART Failure in People With HIV, presented by Michael S. Saag, MD, Constance A. Benson, MD, Monica Gandhi, MD, MPH, Rajesh T. Gandhi, MD, and David H. Spach, MD.

A patient presents to care newly diagnosed with both HIV infection and opioid use disorder (OUD). He is very concerned about his HIV diagnosis but expresses reluctance regarding OUD treatment, stating that he would prefer to focus on HIV treatment first and consider OUD treatment at a later date. Which of the following is true about the effects of maintaining OUD treatment with concurrent HIV treatment?

Responses
A. Improved HIV viral suppression rates are observed only when methadone via directly observed therapy is used for OUD treatment 35 (5%)
B. Improved HIV viral suppression rates are observed only when buprenorphine or naltrexone are used for OUD treatment 94 (13%)
C. Improved HIV viral suppression rates are observed regardless of the medication used for OUD treatment 516 (70%)
D. No effect on HIV viral suppression rates is observed with any of the approved medications for OUD treatment 90 (12%)

Correct answer is C. Improved HIV viral suppression rates are observed regardless of the medication used for opioid use disorder (OUD) treatment. To learn more about the impact of medications used for OUD on HIV outcomes and about integrating medications for OUD into routine HIV care, please register for our upcoming IAS–USA webinar on September 19, 2023, (Part 2 of 6) Substance Use Disorder (SUD) 101: How Should We Care For SUD in Patients With HIV? to be presented by Ellen F. Eaton, MD, and moderated by Sandra A. Springer, MD.

Which of the following best describes the results of a pharmacokinetic study of bictegravir/emtricitabine/tenofovir alafenamide fumarate during pregnancy compared with nonpregnant adults and study participants 12 weeks postpartum?

Responses
A. Bictegravir levels were unchanged 199 (20%)
B. Bictegravir levels were reduced and below estimated levels required to suppress HIV 68 (7%)
C. Bictegravir levels were reduced but still above estimated levels required to suppress HIV 686 (70%)
D. Bictegravir levels were increased, with more treatment-related adverse effects 29 (3%)

Correct answer is C. In this pharmacokinetic study, bictegravir levels in the second and third trimesters greatly exceeded the levels required to inhibit wild-type HIV.

Zhang H, Martin H, Lin L, et al. Pharmacokinetics, safety, and efficacy of bictegravir/emtricitabine/tenofovir alafenamide (B/F/TAF) in virologically suppressed pregnant women with HIV. Poster presented at: IAS 2023, the 12th International AIDS Society Conference on HIV Science; July 24, 2023; Brisbane, Australia.

To learn more about this topic, watch the on-demand webinar, Overview of Data Presented at the 2023 International AIDS Conference presented by Paul A. Sax, MD, and moderated by Marina Klein, MD, MS, on August 29, 2023.

In the open-label extension phase of HPTN (HIV Prevention Trials Network) 084, which of the following best describes the selection of preexposure prophylaxis by the women who participated?

Responses
A. More women chose oral tenofovir disoproxil fumarate (TDF)/emtricitabine (FTC) over injectable cabotegravir (CAB) 231 (18%)
B. More women chose injectable CAB over oral TDF/FTC 830 (66%)
C. There was no clear preference for either treatment option 144 (11%)
D. Most women elected not to participate in the open-label extension 58 (5%)

Correct answer is B. In the open-label extension phase of HPTN (HIV Prevention Trials Network) 084, more women selected injectable cabotegravir (CAB) over oral tenofovir disoproxil fumarate (TDF)/emtricitabine (FTC) for preexposure prophylaxis. To learn more about this topic, register for our free upcoming webinar on August 29, 2023, Overview of Data Presented at the 2023 International AIDS Conference presented by Paul A. Sax, MD, and moderated by Marina Klein, MD, MS.

A 46-year-old man was admitted to the hospital for left-sided weakness and slurred speech. He was found to have an acute right corona radiata ischemic stroke. Laboratory values on admission included a CD4+ count of 127 cells/µL and a viral load of 18,000 copies/mL. He was successfully resuscitated with naloxone after an in-hospital overdose on insufflated fentanyl. What is the next best step?

Responses
A. Refer him to a methadone clinic on discharge 118 (12%)
B. Offer him buprenorphine and naloxone now 514 (55%)
C. Prescribe long-acting naltrexone 122 (13%)
D. None of the above, he is still on morphine for headache after stroke 178 (20%)

Correct answer is B. In this patient, buprenorphine and naloxone are an appropriate treatment strategy. To learn more about this case, and to earn 1.25 CME credits toward the 8 hours of training mandated by the Drug Enforcement Agency (DEA) Medication Access and Training Expansion (MATE) Act, register for our free upcoming webinar on August 22, 2023, (Part 1 of 6) Substance Use Disorder (SUD) 101: Why Should We Care About SUD in Patients With HIV? presented by Sandra A. Springer, MD, and moderated by Ellen F. Eaton, MD.

Register now for the next activity in our free DEA-compliant webinar series:

(Part 2 of 6) Substance Use Disorder (SUD) 101: How Should We Care For SUD in Patients With HIV?
Presenter: Ellen F. Eaton, MD; Moderator: Sandra A. Springer, MD
September 19, 2023

You are providing care for a patient with newly diagnosed HIV infection and opioid use disorder. The patient expresses strong motivation for starting both antiretroviral therapy (ART) and medication for opioid use disorder. You are considering initiating buprenorphine treatment and ART. If recommending an integrase strand transfer inhibitor (InSTI) to use concurrently with buprenorphine, which strategy to manage potential drug-drug interactions is indicated?

 

Responses
A. Reduce buprenorphine dose if using with elvitegravir/cobicistat 122 (19%)
B. Increase dolutegravir to twice daily if using with buprenorphine 34 (5%)
C. Increase buprenorphine dose if using with bictegravir or dolutegravir 29 (4%)
D. No dose adjustments are needed when using any InSTIs with buprenorphine 379 (57%)
E. More information is needed to answer this question 98 (15%)

Correct answer is D. No dose adjustments are needed when using integrase strand transfer inhibitors with buprenorphine.

To learn more about substance use disorders, register for the upcoming webinar on August 22, 2023, (Part 1 of 6) Substance Use Disorder (SUD) 101: Why Should We Care About SUD in Patients With HIV?, presented by Sandra A. Springer, MD, and moderated by Ellen F. Eaton, MD.

A 43-year-old female establishes care for HIV infection. She asks about the human papillomavirus (HPV) vaccine. Which of the following is NOT a potential consideration for offering this vaccine?

Responses
A. Although new HPV infections are most commonly acquired in adolescence and young adulthood, some older adults are at risk. 144 (10%)
B. At any age, a new sex partner is a risk factor for a new HPV infection. 153 (10%)
C. Most sexually active persons have been exposed to some HPV types, but not necessarily to all the HPV types in the vaccine. 218 (14%)
D. The vaccine prevents progression of existing HPV infection. 1,010 (66%)

Correct answer is D. The current HPV vaccine contains 9 serotypes. One randomized, double-blind clinical trial evaluating the efficacy of the quadrivalent HPV vaccine in people with HIV older than 27 years found that the vaccine was safe and immunogenic. However, the trial did not show efficacy for prevention of new anal HPV infections or improvement in anal high-grade squamous intraepithelial lesion outcomes in a population with high levels of prior and current HPV infection.

To learn more about routine and special immunizations for people With HIV, register for the upcoming webinar on August 8, 2023,Routine and Special Immunizations for People With HIV, presented by Steven C. Johnson, MD, and moderated by Melanie A. Thompson, MD.

A 52-year-old man presents for evaluation of newly diagnosed HIV infection. His CD4+ cell count is 125 cells/µL and his HIV viral load is 34,000 copies/mL. Which of the following vaccines would be contraindicated at this time?

Responses
A. Pneumococcal polysaccharide vaccine 44 (9%)
B. Mpox vaccine 38 (7%)
C. Varicella vaccine 353 (70%)
D. Recombinant herpes zoster vaccine 50 (10%)
E. Meningococcal conjugate vaccine 19 (4%)

Correct answer is C. All live, replicating vaccines are contraindicated in people with HIV whose CD4+ cell count is less than 200 cells/µL. These include the measles, mumps, and rubella vaccine, the varicella vaccine, and the yellow fever vaccine. Oral typhoid vaccine and the live attenuated influenza vaccine are also not given to people with HIV regardless of CD4+ cell count and there are alternative vaccines for both diseases. The mpox vaccine is a live vaccine that is nonreplicating and can be safely given at any CD4+ cell count.

To learn more about routine and special immunizations for people With HIV, register for the upcoming webinar on August 8, 2023, Routine and Special Immunizations for People With HIV, presented by Steven C. Johnson, MD, and moderated by Melanie A. Thompson, MD.

Which of the following statements is true during HIV-associated cryptococcal meningitis?

Responses
A. High levels of cerebrospinal fluid (CSF) protein at baseline is associated with lower rates of survival 24% (152)
B. Lower levels of CSF interleukin (IL)-10 is associated with lower rates of survival, regardless of sex 24% (152)
C. Between 40% and 60% of patients have neurocognitive impairment after 12 weeks of treatment 22% (136)
D. High levels of CSF protein is associated with higher CSF fungal burden 30% (85)

Correct answer is B. Lower cerebrospinal fluid (CSF) interleukin (IL)-10 is associated with lower rates of survival survival, regardless of sex, during HIV-associated cryptococcal meningitis.

To learn more about neuropsychiatric complications in people with HIV, see the article, CROI 2023: Neuropsychiatric Complications in People With HIV, by Albert M. Anderson, MD; Beau M. Ances, MD, PhD; Scott L. Letendre, MD; in volume 31.4 of Topics in Antiviral Medicine™

Which of the following groups experienced approximately 4-kg weight loss over 48 weeks after switching from tenofovir alafenamide/emtricitabine/dolutegravir in the ADVANCE (Dolutegravir Plus Two Different Prodrugs of Tenofovir to Treat HIV) study to tenofovir disoproxil/lamivudine/dolutegravir in the CHARACTERISE (a Cross-sectional, Observational Study to Characterise the Transition to Dolutegravir-Based Regimens in South Africa in Terms of the Emergence of Obesity, Viral Re-suppression, and Integration Into Routine Programme Care) study?

Responses
A. Men 18% (101)
B. Women 46% (259)
C. Both men and women 36% (205)

Correct answer is B. Women lost approximately 4 kg over 48 weeks after the switch to tenofovir disoproxil/lamivudine/dolutegravir in the CHARACTERISE (a Cross-sectional, Observational Study to Characterise the Transition to Dolutegravir-Based Regimens in South Africa in Terms of the Emergence of Obesity, Viral Re-suppression, and Integration Into Routine Programme Care) study, but a similar weight effect was not observed in men.

To learn more about metabolic and other complications of HIV Infection, see the article, CROI 2023: Metabolic and Other Complications of HIV Infection, by Sudipa Sarkar, MD; Todd T. Brown, MD, PhD, in volume 31.4 of Topics in Antiviral Medicine™

TRUNCATE-TB (Two-Month Regimens Using Novel Combinations to Augment Treatment Effectiveness for Drug-Sensitive Tuberculosis), a novel strategy study that aimed to reduce tuberculosis treatment duration for some individuals, showed which of the following regimens to be noninferior to the standard 6-month regimen?

Responses
A. Bedaquiline, linezolid, isoniazid, pyrazinamide, and ethambutol for 8 weeks 26% (33)
B. Bedaquiline, linezolid, isoniazid, pyrazinamide, and ethambutol initially for 8 weeks and extended by at least 1 month if still symptomatic and smear-positive 34% (43)
C. High-dose rifampicin, linezolid, isoniazid, pyrazinamide, and ethambutol for 8 weeks 20% (25)
D. High-dose rifampicin, linezolid, isoniazid, pyrazinamide, and ethambutol initially for 8 weeks and extended by at least 1 month if still symptomatic and smear-positive 20% (25)

Correct answer is B. Bedaquiline, linezolid, isoniazid, pyrazinamide, and ethambutol initially for 8 weeks and extended by at least 1 month if still symptomatic and smear-positive was noninferior to the standard 6-month regimen for tuberculosis treatment.

To learn more about tuberculosis and infectious complications in persons With HIV, see the article, CROI 2023: Tuberculosis and Infectious Complications in Persons With HIV, by Andrew D. Kerkhoff, MD, PhD, MSc, and Diane V. Havlir, MD, in volume 31.4 of Topics in Antiviral Medicine™

A 35-year-old female fully vaccinated runner with hypertension presents 7 months after initial mild COVID-19 infection for dizziness and palpitations. She reports palpitations and dizziness when getting up from a seated or lying position as well as with mild exertion. Symptoms are associated with generalized weakness, post-exertional malaise, forgetfulness, difficulty concentrating, headaches, and blurred vision. The results of a comprehensive physical examination are within normal limits with the exception of tachycardia upon rising from a lying position. Laboratory test values including complete blood cell count, comprehensive metabolic panel, and D-dimer were within normal limits with the exception of C-reactive protein level of 15 mg/dL. Tilt table testing is positive. Which of the following is the next best step in management?

Responses
A. Start rivaroxaban 11% (41)
B. Increase water and salt intake 22% (82)
C. Initiate midodrine 10% (38)
D. Order lower extremity compression garments 19% 65
E. Start a rehabilitation plan that focuses on upright training techniques 38% 139

Correct answer is D. Postural orthostatic tachycardia syndrome (POTS) is a well-characterized sequela of COVID-19. Although clinical trials are limited in POTS, observational data support pharmacologic and non-pharmacologic management, including use of lower extremity compression garments, increasing water and salt intake, rehabilitation with recumbent training techniques, and initiation of midodrine, pyridostigmine, fludrocortisone, beta blockers, clonidine, or modafinil. In a patient with hypertension, however, use of lower extremity compression garments is the best option listed.

To learn more about the long COVID diagnosis and management, see the presentation, Evaluation and Management of Long COVID: A Case-Based Panel Discussion, presented by Tiffany Walker, MD, with the panelists: Annie Antar, MD, PhD; Judith S. Currier, MD; Carlos del Rio, MD; Roger Paredes, MD, PhD; Jeffrey N. Siegelman, MD; Kanecia Zimmerman, MD, MPH.

A 41-year-old woman with no past medical history presents 4 months after a mild SARS-CoV-2 infection with primary complaint of shortness of breath. Over the past 3 months she has noticed that her heart races and she feels dizzy while getting up to walk. This occurs often, but not every time she exerts herself. During these spells, she feels short of breath. Otherwise, she has no difficulty breathing. She was previously able to run 10 miles, but now is having difficulty running 1 mile. On examination, her vitals are normal with no orthostatic hypotension. A cardiovascular exam reveals regular rate and rhythm (RRR), no jugular venous pressure (JVP) elevation, and no peripheral edema. Lung exam is clear to auscultation bilaterally (CTAB). Electrocardiogram (EKG) is within normal limits. What is the appropriate next diagnostic test?

Responses
A. Transthoracic echocardiogram
28% (144)
B. Chest computed tomography (CT) scan with contrast 19% (98)
C. Tilt table testing 32% (166)
D. Cardiac magnetic resonance imaging (MRI) 5% 29
E. Ventilation/perfusion scan 16% 82

Correct answer is C. Postural orthostatic tachycardia syndrome (POTS) is a form of dysautonomia that is characterized by dyspnea, palpitations, and dizziness upon changes in posture. POTS has been reported to occur in up to 30% of long COVID patients, most commonly occurring in younger and female populations. It can be mimic for cardiopulmonary sequelae of COVID-19, as it shares similar symptom profiles. Common POTS symptoms, as well as associated symptoms of headaches, chest pain, brain fog, and sleep disturbances, can lead to substantial disability in the affected population. Appropriate diagnostic management includes a comprehensive cardiopulmonary physical exam, including orthostatic vitals to rule out orthostatic hypotension, followed by tilt table testing. A 30 beats/min or greater increase in heart rate without orthostatic hypotension within the first 10 minutes of a head-up tilt table test is 92% specific for the diagnosis of POTS.

To learn more about the the clinical relationship between long COVID and HIV, watch the presentation, Long COVID: Diagnosis and Management, presented by Tiffany Walker, MD.

In the United States, are people with HIV more likely to experience persistent symptoms and incident diagnoses (eg, long COVID) than people who are HIV negative?

Responses
A. Yes
44% (379)
B. No 13% (113)
C. It is unclear. Similarly sized studies have conflicting results 43% (369)

Correct answer is A. Smaller studies published in 2022 have conflicting results on this question. Notably, Peluso and Antar1 found in an unadjusted analysis of fewer than 100 people that HIV is a risk factor for long COVID. However, in the same cohort with a larger sample size and adjustment, HIV was not found to be a risk factor for long COVID overall, though it was a risk factor for neurocognitive long COVID.2

An electronic medical record study3 of approximately 28,000 people with HIV and approximately 28,000 propensity-matched (for age, sex, comorbidities, etc) people without HIV demonstrates definitively that HIV is a risk factor for long COVID, with increased odds ratios between 2 and 3 for various post-acute symptoms such as fatigue, cognitive impairment, and body aches and for incident diagnoses such as thrombotic disease, diabetes, and mental disorders.

1. Peluso MJ, Antar AAR. Long COVID in people living with HIV. Curr Opin HIV AIDS. 2023;18(3):126-134.
2. Peluso MJ, Deveau TM, Munter SE, et al. Chronic viral coinfections differentially affect the likelihood of developing long COVID. J Clin Invest. 2023;133(3): e163669.
3. Yendewa G, Perez JA, Patil N, et al. HIV infection is associated with higher risk of post-acute sequelae of SARS-CoV-2 (PASC) however vaccination is protective. SSRN. Preprint posted online November 15, 2022. http://dx.doi.org/10.2139/ssrn.4276609.

To learn more about the the clinical relationship between long COVID and HIV, register for the upcoming virtual course , State-of-the-Art Update on Long COVID: Pathogenesis, Management, Clinical Trial Updates, and Patient Perspective, on June 22, 2023.

After an annual 2% to 3% decline in new HIV diagnoses in the US in the several years prior to COVID-19, there was a 17% reduction in the number of new HIV diagnoses in 2020 compared with 2019. Which of the following is understood to be the cause of the decline in new HIV diagnoses?

Responses
A. The decline was likely the result largely of reduced testing during the COVID-19 pandemic 43% (256)
B. The decline was likely the result largely of behavior change (eg, social distancing, fewer sexual partners) 6% (36)
C. The decline was likely a result of both reduced testing and substantial behavior changes 37% (225)
D. There are inadequate data to determine the cause of decreased numbers of new diagnoses 14% (82)

Correct answer is A. Investigators from the Centers for Disease Control and Prevention used 3 different math models to determine that reduced HIV testing accounted for the reduction in new HIV diagnoses.

To learn more about the latest research on the epidemiology of HIV and other sexually transmitted infections, see the article, CROI 2023: Epidemiologic Trends and Prevention for HIV and Other Sexually Transmitted Infections, written by Albert Y. Liu, MD, MPH, and Susan P. Buchbinder, MD.

Which of the following pre-exposure prophylaxis (PrEP) regimens are recommended for coverage for missed visits for injectable cabotegravir

Responses
A. Daily oral tenofovir disoproxil fumarate/emtricitabine (TDF/FTC) 46% (195)
B. Daily oral tenofovir alafenamide/emtricitabine (TAF/FTC) 13% (57)
C. Daily oral cabotegravir (CAB) 30% (127)
D. On-demand TDF/FTC 11% (45)

Correct answer is A. The prescribing information for cabotegravir recommends oral cabotegravir for up to 2 months after a missed injection visit. After 2 months, an alternative form of PrEP should be used. Oral cabotegravir has not been studied as an oral PrEP option. Based on the data for daily oral TDF/FTC, as well as cost and implementation considerations, TDF/FTC would be the best option for covering missed injection visits.

To learn more about the current strategies in the prevention of HIV, see the presentation, Latest Strategies in the Prevention of HIV, by Hyman Scott, MD, MPH.

A 47-year-old man presents with right ankle swelling and migratory polyarthritis. He is sexually active with men and women. His exam reveals right ankle effusion and distal extremity necrotic skin lesions. Which one of the following is NOT true with regard to disseminated gonococcal infection (DGI)?

Responses
A. The highest diagnostic yield of a microbiologic diagnosis is culture of the joint fluid 43% (241)
B. There is a recent shift in the epidemiology with a male predominance 12% (65)
C. The most frequent clinical manifestation is oligoarticular arthritis 17% (98)
D. Terminal complement deficiency is a risk factor for recurrent infection 11% (64)
E. Mucosal sites of infection are frequently asymptomatic 17% (99)

Correct answer is A. DGI is uncommon, estimated to account for 0.5% to 3% of gonococcal infections. Historic data described that DGI was seen more frequently in females, but recent series have demonstrated a shift to male predominance. The clinical presentation can include an oligoarticular septic arthritis or a tenosynovitis/dermatitis syndrome. The highest diagnostic yield of a microbiologic diagnosis is a mucosal site of exposure, which is often asymptomatic.

To learn more about the current epidemiology, clinical presentations, and management of the most common sexually transmitted infections (STIs) in adults with HIV infection, see the presentation, Treatment and Management of Syphilis and Gonorrhea, by Kimberly A. Workowski, MD.

Should people with HIV routinely be vaccinated against meningococcus B?

Responses
A. Yes
61% (375)
B. No 39% (236)

Correct answer is B. The United States Department of Health and Human Services (DHHS) Guidelines for the Prevention and Treatment of Opportunistic Infections in Adults and Adolescents with HIV do not recommend routine meningococcal B vaccination for people with HIV (in contrast to meningococcal vaccine covering the A, C, W, and Y serotypes). If a person with or without HIV is at risk due to a local outbreak, meningococcal B vaccination would be recommended. Other risk factors (eg, complement deficiency) will trigger meningococcal B vaccination but not HIV infection itself.

To learn more about the implementing primary care practices, including vaccination schedules, see the presentation, Primary Care for People with HIV: Immunizations , by Monica Gandhi, MD, MPH.

Which of the following statements about long-acting injectable cabotegravir (CAB-LA) is FALSE?​

Responses
A. No HIV infections have occurred as long as CAB-LA is administered on time, only when doses are missed.​ 38% (158)
B. Integrase strand transfer inhibitor (InSTI) resistance has emerged within the first 6 months of a breakthrough infection.​ 18% (73)
C. No InSTI resistance has emerged more than 6 months after CAB-LA was administered.​ 21% (89)
D. The US Food and Drug Administration and Centers for Disease Control and Prevention recommend HIV RNA screening during CAB-LA preexposure prophylaxis. 23% (93)

Correct answer is A. There were 6 breakthrough infections in the HPTN 083 study, despite on-time injections.

To learn more about the latest strategies for prevention of HIV and sexually transmitted diseases, see the presentation, The ART of HIV and STI Prevention: An Update From CROI 2023 , by Susan P. Buchbinder, MD.

A female patient is taking dolutegravir + emtricitabine/tenofovir alafenamide fumarate (DTG + FTC/TAF) and has gained 20 lb over 2 years on therapy. Based upon the CHARACTERISE (a Cross-sectional, Observational Study to Characterise the Transition to Dolutegravir-based Regimens in South Africa in Terms of the Emergence of Obesity, Viral Re-suppression and Integration Into Routine Programme Care) study, which of the following can you tell her may happen to her weight if she switches to dolutegravir + emtricitabine/tenofovir disoproxil fumarate (DTG + FTC/TDF)?

Responses
A. Gain 9% (63)
B. Lose 53% (358)
C. No change 28% (188)
D. There are no data to inform this question 10% (66)

Correct answer is B. CHARACTERISE showed that women who were randomly assigned to DTG + FTC/TAF in the ADVANCE (Dolutegravir plus Two Different Prodrugs of Tenofovir to Treat HIV) trial and were then switched during the open-label phase to dolutegravir + lamivudine/tenofovir disoproxil fumarate (DTG + 3TC/TDF) experienced weight loss.

To learn more about the latest antiretroviral management strategies for HIV, see the presentation, State of Antiretroviral Management of HIV from CROI 2023 , by Eric S. Daar, MD.

A 22-year-old transgender woman presents to the emergency department for evaluation of a possible bacterial STI. She is using gender-affirming hormone therapy (GAHT). Her psychosocial history is significant for trauma, history of injection drug use, and housing instability. Which of the following statements regarding her possible preexposure prophylaxis (PrEP) options is true?

Responses
A. She is not eligible for PrEP, as drug-drug interactions may contraindicate the use of all PrEP and GAHT 3% (18)
B. Only long-acting injectable PrEP is appropriate because of her housing instability and trauma history 4% (24)
C. Only oral PrEP is appropriate, as we do not yet have enough data on the use of long-acting injectable cabotegravir PrEP among those who use injection drugs 9% (56)
D. She should be counseled on the various advantages and disadvantages of each delivery method and supported in the use of either product 84% (503)

Correct answer is D. GHAT does not affect cabotegravir pharmacokinetics. However, formal drug-drug interaction studies have not evaluated cabotegravir’s effects on GAHT. Mechanistically, there are no anticipated interactions, and the effects on hormonal contraceptive agents are not clinically significant. Although we require additional data on the effective implementation of long-acting injectable PrEP among those who use injection drugs, injection drug use is not a contraindication to long-acting injectable PrEP. Sexual transmission still accounts for most HIV transmission among injection drug users. Effective PrEP counseling should be client-centered, focus on empowerment, and allow individuals to make the best and most informed choices around HIV prevention for themselves.

To learn more about gender-reaffirming hormone therapy (GAHT), see the presentation, THE BIDIRECTIONAL EFFECTS OF HORMONE THERAPY AND PREP IN TRANSGENDER INDIVIDUALS , by Jill Blumenthal, MD, MAS.

Which statement is correct about how mRNA technology can uniquely assist in the development of an HIV vaccine?

Responses
A. Allow the production of immunogens that would otherwise not be possible 20% (133)
B. Enable several HIV variant immunogens to be administered 21% (137)
C. Allow for a more rapid evaluation of HIV immunogens 32% (210)
D. Induce broadly neutralizing antibodies (bNAbs) 27% (171)

Correct answer is C. mRNA can be rapidly modified and produced for vaccine development and is faster than any other current technology. There are many ways to produce immunogens that are not unique to the mRNA platform, including inactivation of viral particles and recombinant protein production. Other vaccine platforms can administer several immunogens at one time (eg, Prevnar-20). There is no technologic advance in vaccine synthesis itself that will induce bNAbs (including mRNA). The HIV field is slowly learning how to mutate immunogen sequences to hopefully achieve this goal.

To learn more about the latest mRNA Vaccine Technology and the Prospects for an HIV Vaccine, see the presentation, mRNA Vaccine Technology and the Prospects for an HIV Vaccine, by Paul Goepfert, MD.

When is the highest risk period for emergence of drug resistance with long-acting cabotegravir?

Responses
A. During oral lead-in 38% (60,309)
B. During injection period with on-time injections 31% (49,322)
C. Daily injection period with missed injection 30% (49,217)
D. During pharmacokinetic (PK) tail 1% (1,215)

Correct answer is B. Although long-acting cabotegravir was superior to tenofovir disoproxil fumarate/emtricitabine (TDF/FTC) in HIV prevention clinical trials, HIV seroconversions in HPTN (HIV Prevention Trials Network) 083 did occur during the oral lead-in, injection phase, and PK tail phase. HIV seroconversions that occurred in the setting of on-time injection were often identified late secondary to low viral load and delayed seroconversion, and were associated with emergence of drug-associated resistance mutations.

To learn more about the latest strategies in the prevention of HIV, see the presentation, Latest Strategies in the Prevention of HIV, by Hyman Scott, MD, MPH.

A 48-year-old transgender woman presents seeking gender-affirming care. Past medical history includes HIV, diabetes mellitus type II, and hyperlipidemia. Her medications include rosuvastatin 10 mg daily, metformin 500 mg twice a day, and bictegravir/emtricitabine/tenofovir alafenamide. She currently smokes 15 cigarettes per day. What should you consider when starting gender-affirming hormone therapy?

Responses
A. Start estrogen and spironolactone regardless of smoking status 3% (24)
B. Start transdermal estrogen and spironolactone along with smoking cessation counseling 68% (512)
C. Do not begin hormone therapy until the patient has stopped smoking 16% (119)
D. Begin spironolactone only 13% (99)

Correct answer is B. Tobacco use combined with estrogen therapy is associated with increased venous thromboembolic risk. All transgender women who smoke should be counseled on the risks of tobacco and provided with smoking cessation options. Many transgender women may be unable or unwilling to quit smoking. Cigarette smoking alone is not an absolute contraindication to starting estrogen therapy. Following an in-depth and informed consent discussion, it is reasonable to prescribe estrogen using a harm reduction approach, with a recommended route of transdermal estrogen.

To learn more about the latest in gender-affirming hormone therapy, see the newest release in Topics in Antiviral Medicine™, Providing Gender-Affirming Care to Transgender and Gender-Diverse Individuals With and at Risk for HIV.

Heart disease in PWH results from increased inflammation and residual immune activation. Which of the following statements is true?

Responses
A. Initiation of ART results in increased immune activation in PWH 17% (106)
B. Immune activation decreases but persists after initiation of ART 76% (472)
C. Nadir CD4+ cell count is unrelated to immune activation and its effects on cardiovascular disease (CVD) 7% (45)

Correct answer is B. Immune activation decreases but persists at a level above that seen in healthy controls after ART initiation. Nadir CD4+ cell count, reflective of the immune set point, is related to CVD in PWH.

To learn more about the latest in cardiovascular disease and HIV, see Dr Steven K. Grinspoon’s presentation from Scott M. Hammer Annual Update on HIV Management in New York, New York.

When is the highest risk period for developing drug resistance with long-acting cabotegravir (LA CAB) when used for prevention?

Responses
A. They have been commonly used for decades, with more than 100 trials conducted pre-COVID-19 20% (70)
B. They are larger than a combination of individually randomized clinical trials (RCTs) answering the same questions. 28% (98)
C. Sites may have the option to include or exclude participants for each treatment based on product availability 14% (50)
D. Sites may have the option to include or exclude participants for each treatment based on product availability 12% (42)
E. To maintain the blinding, only agents with the same mode of administration (eg, oral, intravenous) can be combined into a single platform trial 26% (94)

Correct answer is D. In platform trials, sites may have the option to include or exclude participants for each treatment based on product availability or product contraindications.

Platform trials are relatively new, with less than 2 dozen pre-COVID-19. Platform trials are more efficient than a series of individual RCTs with smaller sample sizes. Platform trials do not need longer consent forms than those for individual RCTs. Platform trials can accommodate arms with different modes of administration.

Webcasts from CROI 2023 will be available to the public on March 22. Keep an eye on the News and Announcement page for the latest updates.

Visit our CROI website here.

When is the highest risk period for developing drug resistance with long-acting cabotegravir (LA CAB) when used for prevention?

Responses
A. During oral lead-in 8% (63)
B. During the injection period with on-time injections 16% (123)
C. During the injection period with missed injections 46% (349)
D. During the PK tail 30% (227)

Correct answer is B. Although LA CAB was superior to tenofovir disoproxil fumarate/emtricitabine (TDF/FTC) in HIV prevention clinical trials, HIV seroconversions in HPTN 083 did occur during the oral lead-in, injection phase, and PK tail phase. HIV seroconversions that occurred in the setting of on-time injections were often identified late due to low viral load and delayed seroconversion and were associated with the development of drug-associated resistance mutations.

True or False: In the event of an mpox outbreak, vaccination with the third-generation live modified vaccinia Ankara (MVA) should first be administered in a post-exposure ring to exposed individuals, then rolled out to at-risk individuals (multipartnered men who have sex with men [MSM]), regardless of exposure as a preventive strategy.

Responses
A. True 74% (363)
B.  False 26% (130)

Correct answer is A. True. Best practice outbreak prevention includes initial highly targeted ring vaccination and then broader access to at-risk community members.

Webcasts from CROI 2023 will be available to the public on March 22. Keep an eye on the News and Announcement page for the latest updates.

True or False: Viral and symptom rebound in SARS-CoV-2 infection are associated exclusively with nirmatrelvir (Paxlovid) use

Responses
A. True 12% (85)
B.  False 88% (635)

Correct answer is B. False. Viral and Symptom rebound in SARS-CoV-2 infection can happen without nirmatrelvir (Paxlovid) use.

Webcasts from CROI 23 will be available to the public on March 22. Keep an eye on the News and Announcement page for the latest updates.

What does the passing of the Mainstreaming Addiction Treatment Act (MAT Act) and the Medication Access and Training Expansion Act (MATE Act) mean for clinicians?

 

Responses
A. Clinicians no longer need a special X-waiver to prescribe buprenorphine to treat opioid use disorder, a regular DEA number and controlled substance license will suffice 58% (183)
B. Clinicians still need a special X-waiver, but there is no limit to the number of patients they can prescribe buprenorphine to 6% (19)
C. If clinicians have a controlled substance license to prescribe schedule II, III, IV or V medications then they must take a yearly 8-hour training course on the treatment and management of patients with opioid or other substance use disorders when the law goes into effect 21% (64)
D. None of the above 15% (47)

Correct answer is A. Clinicians no longer need a special X-waiver to prescribe buprenorphine to treat opioid use disorder, a regular DEA number and controlled substance license will suffice. Clinicians with a controlled substance license to prescribe schedule II, III, IV or V medications are required to take a 1-time only 8-hour training course on the treatment and management of patients with opioid or other substance use disorders on or 180 days after June 21, 2023, when the law goes into effect, and there is no limit to the number of patients they can prescribe buprenorphine to.

To learn more about this update to legislation and how to manage opioid use disorder in people with HIV, register for the Scott M. Hammer Annual Update on HIV Management in New York, New York, or the Annual Update on HIV Management in Atlanta, Georgia.

Which class of diabetes medications is preferred for a patient with a history of heart failure?

Responses
A. Sulfonylureas 5% (26)
B. Thiazolidinediones 6% (29)
C. GLP-1 receptor agonists 22% (112)
D. DPP-IV inhibitors 6% (31)
E. SGLT2 inhibitors 61% (315)

Correct answer is E. SGLT2 inhibitors have been shown to decrease the risk of heart failure hospitalizations in both systolic and diastolic dysfunction. As a result, these drugs are preferred in patients with diabetes who also have heart failure, unless there is a contraindication.

To learn more about this topic, click the link below to watch, Management of Diabetes in People With HIV, presented by Todd T. Brown, MD, PhD.

A 45-year-old woman with HIV was diagnosed in 2005 and started antiretroviral therapy (ART) in 2012. Her CD4+ count at diagnosis was 245 cells/µL. She now has controlled hypertension, an HIV RNA count of fewer than 50 copies/mL, a current CD4+ count of 560 cells/µL, and hepatitis C that she will soon begin to treat. Her ACC/AHA (American College of Cardiology/American Heart Association) Pooled Cohort Equation score is 7.0%. Which of the following factors does NOT affect your decision about whether she needs a statin?

Responses
A. Current undetectable viral load 55% (138)
B. Nadir CD4+ of 245 cells/µL 28% (70)
C. Prolonged viremia in the past 17% (43)

Correct answer is A. A currently undetectable viral load does not compensate for a history of prolonged uncontrolled viremia, which, like B and C, are HIV risk-enhancing factors that increase the risk of cardiovascular disease.

To learn more about this topic, click the link below to watch, Aging and HIV, presented by Melanie Thompson, MD, at the September 8 course in Los Angeles.

A 50-year-old man with HIV is diagnosed with an anal high-grade squamous epithelial lesion by high-resolution anoscopy (HRA) and biopsy. What treatment would you recommend?

Responses
A. Observation only and repeat HRA every 6 months 8% (20)
B. Surgery 27% (73)
C. Radiatio 8% (23)
D. Hyfrecation (office-based electrocautery/ablation) 57% (155)

Correct answer is D. The ANCHOR (Anal Cancer HSIL Outcomes Research) study demonstrated a decrease in anal cancer incidence when lesions were treated versus observation only.

To learn more about this topic, click the link below to watch Anal Cancer Screening: What Now?, presented by Susan Cu-Uvin, MD, at The IAS–USA 30th Annual Virtual Update on HIV Management in Chicago, Illinois, on December 8, 2022.

Which of the following statements is true?

 

Responses
A. Tenofovir disoproxil fumarate and tenofovir alafenamide have different drug resistance profiles 8% (22)
B. The cytidine analogs lamivudine and emtricitabine are equally effective at treating viruses with M184V as with wild-type viruses 6% (44)
C. Dolutegravir resistance mutations occur more commonly in InSTI-naive, nRTI-experienced persons receiving dolutegravir than in ART-naive persons receiving a first-line dolutegravir-containing regimen 41% (114)
D. Boosted lopinavir, atazanavir, and darunavir have similar genetic barriers to resistance 14% (38)
E. InSTI resistance mutations emerge in about 1% of persons per year receiving a first-line dolutegravir- or bictegravir-containing regimen 31% (87)

Correct answer is C. Dolutegravir resistance mutations have rarely been reported in ART-naive persons. They have occurred in about 1% to 4% of InSTI-naive and nRTI-experienced persons receiving dolutegravir over a period of 1 to 2 years. Option A is false because tenofovir disoproxil and tenofovir alafenamide are affected by the same mutations. Option B is false because lamivudine and emtricitabine reduce plasma HIV-1 RNA levels by about 0.5 logs when treating viruses with M184V, but reduce virus levels by almost 2.0 logs when they treat wild-type viruses. Option D is false because darunavir has the highest genetic barrier to resistance, followed by lopinavir/ritonavir and then atazanavir/ritonavir. Atazanavir/ ritonavir is better tolerated than lopinavir/ritonavir and may lead to better outcomes in some circumstances. Option E is false because InSTI resistance mutations rarely emerge in persons receiving first-line dolutegravir- or bictegravir-containing regimens.

To learn more about this topic, click the link below to watch the December 14, 2022, webinar, Basics of HIV Drug Resistance Mutations, presented by Francesca Ceccherini Silberstein, PhD, Douglas D. Richman, MD, and Robert W. Shafer, MD.

You have a patient with HIV who is considering pregnancy and would like more information on breastfeeding. You inform them that the risk of HIV transmission via breast milk from a breastfeeding person on antiretroviral therapy (ART) with a consistently undetectable viral load is which of the following?

Responses
A. 20%
5% (45)
B. 10% 4% (44)
C. 5% 9% (95)
D. <1% 82% (820)

Correct answer is D. The risk of HIV transmission via breastfeeding was about 16% prior to the availability of ART. With viral suppression on ART, the transmission rate is under 1% (0.3% over 6 months and 0.6% over 12 months in a low-resource setting in the study by Flynn).

To learn more about this topic, click the button below to register for the January 17 webinar, Breast/Chestfeeding Among Individuals With HIV: Where Have We Been and Where Are We Going?, presented by Judy Levison, MD, MPH.

Which of the following statements related to HIV and Alzheimer’s disease is correct?

 

Responses
A. Cerebrospinal fluid (CSF) biomarkers of Alzheimer’s disease are perturbed in the setting of HIV
36% (321)
B. Brain structural imaging can distinguish HIV-related cognitive impairment from that of Alzheimer’s disease in the clinical setting 17% (153)
C. Lewy body dementia is more common than Alzheimer’s disease among people with HIV 20% (181)
D. None of the above 27% (248)

Correct answer is A. CSF biomarkers of Alzheimer’s disease are perturbed in the setting of HIV. There are data showing that about 1/3 of people getting Alzheimer’s disease biomarkers tests in CSF have ‘borderline’ reports due to perturbations of either amyloid or tau. Unfortunately, brain imaging is a poor distinguishing feature between HIV-related impairment and Alzheimer’s disease at the bedside. Using advanced machine learning techniques, we can teach a computer to categorize cases by disease, but we don’t see these well in personally reviewing images, and there is too much overlap in changes for it to be useful in such a setting. We don’t know the frequency of Lewy Body Dementia or Alzheimer’s disease in people living with HIV. However, it is highly unlikely that the frequency will be so different in people living with HIV compared to those without infection. So, Alzheimer’s is very likely to be more common than Lewy Body Dementia.

To learn more about this topic, click the button below to watch the webinar, Addressing Challenges in Alzheimer’s Disease in People With HIV, presented by Victor G. Valcour, MD, PhD.

A 65-year-old male presents with low-grade fever, cough, headache, and sore throat for 2 days. Rapid COVID-19 test is positive at home; rapid influenza test is negative. His HIV RNA is less than 20 copies/mL and his CD4+ count is 560 cells/µL. He smokes cigarettes and medications include dolutegravir (DTG)/ lamivudine (fdc), rosuvastatin (for hyperlipidemia). His oxygen saturation is between 94% and 95% on room air. How would you treat his SARS-CoV-2 infection?

Responses
A. Initiate nirmatrelvir/ritonavir 58% (259)
B. Administer bamlanivimab/ etesevimab infusion 3% (16)
C. Administer bebtelovimab infusion 2% (9)
D. Administer remdesivir outpatient infusion (over 3 days) 18% (83)
E. Initiate molnupiravir 10% (42)
F. Initiate prednisone (40 mg daily 9% (38)

Correct answer is A. Nirmatrelvir/ritonavir is the treatment of choice for most non-hospitalized patients with mild to moderate COVID-19. Statins should be held during the treatment course. Anti-SARS-CoV-2 monoclonal antibodies (options B and C) are no longer active against the dominant circulating variants. Remdesivir is logistically more difficult to administer than nirmatrelvir/ritonavir but should be considered if drug interactions preclude use of nirmatrelvir/ritonavir. Molnupiravir appears to be less efficacious than nirmatrelvir/ritonavir and remdesivir.

To learn more about this topic, click the link below to watch, Recent Co-Infections in the Setting of ART: mpox and COVID-19, presented by Constance A. Benson, MD, at the December 15 course, The 2022 IAS-USA Guidelines Virtual Update: Antiretroviral Drugs for Treatment and Prevention of HIV Infection in Adults.

A 59-year-old man comes to your office to discuss starting preexposure prophylaxis (PrEP). He has a history of kidney disease, with a creatinine clearance of 40 mL/minute. Which of the following PrEP options should be avoided?

Responses
A. Cabotegravir long-acting injections 10% (30)
B. Tenofovir disoproxil fumarate/emtricitabine daily oral pills 74% (217)
C. Tenofovir alafenamide/emtricitabine daily oral pills 16% (47)

Correct answer is B. Daily oral tenofovir disoproxil fumarate/emtricitabine should be avoided in people with creatinine clearance of less than 60 mL/minute. The other options are reasonable in people with creatinine clearance of 30 to 60 mL/minute.

To learn more about this topic, click the link below to watch, HIV Prevention and STI Screening and Prevention, presented by Raphael J. Landovitz, MD, at the December 15 course, The 2022 IAS-USA Guidelines Virtual Update: Antiretroviral Drugs for Treatment and Prevention of HIV Infection in Adults.

You are providing care for a patient with newly diagnosed HIV infection and opioid use disorder (OUD). The patient expresses strong motivation for starting both antiretroviral therapy (ART) and medication treatment for OUD and notes a preference for a self-administered option that he can take on his own. You are considering initiating buprenorphine treatment and ART. If recommending an integrase strand transfer inhibitor (InSTI) to use concurrently with buprenorphine, which strategy to manage potential drug-drug interactions is indicated?

Responses
A. Reduce buprenorphine dose if using with elvitegravir (EVG)/cobicistat (COBI) 24% (72)
B. Increase dolutegravir (DTG) to twice daily if using with buprenorphine 10% (31)
C. Increase buprenorphine dose if using with bictegravir (BIC) or DTG 5% (16)
D. No dose adjustments are needed when using any of the InSTIs with buprenorphine 61% (187)

Correct answer is D. Buprenorphine does not interact with InSTIs; therefore, no dose adjustments are necessary.

To learn more about this topic, click the link below to watch, Substance Use Disorder, presented by Ellen Eaton, MD, and Sandra Springer, MD, at the December 15 course, The 2022 IAS-USA Guidelines Virtual Update: Antiretroviral Drugs for Treatment and Prevention of HIV Infection in Adults.

A 20-year-old woman with vaginal discharge is seen by her ob-gyn. She reports 8 new male sexual partners in the last 6 months. She engages in vaginal and anal sex, and condoms are used sporadically. She has been on oral contraception for a few years but admits to missing several doses each month. Upon further questioning, she reports increased alcohol use and challenges with her parents at home, and at school. She is afraid of sexually transmitted infections (STIs), especially HIV. She was diagnosed with chlamydia using a urine DNA probe but had a negative HIV serology. Unexpectedly, her urine beta-human chorionic gonadotropin (HCG) came back positive. She wants to discuss ways to decrease her HIV risk, particularly now that she is pregnant. TDF/FTC can be safely used during pregnancy; however, it requires daily dosing and may not be ideal for this patient. Which of the following statements regarding the potential use of antibodies in HIV prevention is FALSE?

Responses
A. VRC01 was tested in a phase IIb study involving young women in sub-Saharan Africa and was effective in preventing new infections compared with placebo 31% (224)
B. Resistance of circulating viral strains is a main challenge in the development of anti-HIV monoclonal antibodies for prevention 18% (130)
C. The AMP (Antibody-Mediated Prevention) studies showed that a neutralizing antibody can protect from infection with antibody-sensitive viral strains 10% (75)
D. Antibody combinations are likely needed to achieve prevention efficacy 12% (93)
E. Antibodies have potential advantages over antiretrovirals, including their long half-lives and safety profile. These characteristics may be particularly attractive to sub-populations at risk of acquiring HIV 29% (211)

Correct answer is A. Results from 2 parallel phase IIb studies (AMP studies) that tested the efficacy of the anti-HIV-1 broadly neutralizing antibody (bNAb) VRC01 against HIV acquisition among individuals at risk were published recently. The 2 studies enrolled more than 3,000 individuals, and about 30,000 infusions of VRC01 were administered. The antibody infusions were very well tolerated and gives support to the notion that passive transfer of antibodies is a safe strategy. Despite showing the expected safety and pharmacokinetic (PK) profiles of VRC01, the studies did not meet the primary endpoint of efficacy when the active and placebo groups were compared. However, sub-analysis of prevention efficacy according to virus sensitivity to VRC01 demonstrated efficacy of more than 75% in preventing infection from viruses that were highly sensitive to the antibody. Although this observation provides proof of concept that prevention can be achieved by the passive transfer of antibodies, it highlights the need to improve the strategy by combining antibodies of different specificity or developing new antibodies with a broader range of antiviral activity.

To learn more about this topic, click the link below to watch, bNAbs: Where Are We Going?, presented by Marina Caskey, MD, at the December 8 virtual Chicago course.

After failing a regimen of tenofovir disoproxil fumarate (TDF)/lamivudine/efavirenz that resulted in resistance mutations in the nucleos(t)ide components, which of these regimens has been shown to be least effective?

Responses
A. Boosted darunavir with zidovudine/lamivudine 15% (120)
B. Boosted darunavir with TDF/lamivudine 14% (108)
C. Dolutegravir with zidovudine/lamivudine 10% (73)
D. Dolutegravir with TDF/lamivudine 11% (89)
E. None of the regimens above were inferior 50% (394)

Correct answer is E. The NADIA (Nucleosides And Darunavir/Dolutegravir In Africa) trial randomly assigned 928 patients experiencing virologic failure on TDF/lamivudine/efavirenz to 1 of the 4 regimens listed in options A through D. None of the 4 regimens were statistically inferior in that study, although both the World Health Organization (WHO) guidelines and some experts originally believed otherwise.

To learn more about this topic, click the link below to register for our December 14 webinar on the Basics of HIV Drug Resistance Mutations, which will highlight the IAS–USA 2022 Update of the Drug Resistance Mutations in HIV-1, released on September 23, 2022.

A 48-year-old transgender woman presents seeking gender-affirming care. Past medical history includes HIV, diabetes mellitus type II, and hyperlipidemia. Her medications include rosuvastatin 10 mg daily, metformin 500 mg twice daily, and bictegravir/emtricitabine/tenofovir alafenamide. She currently smokes 15 cigarettes per day. What should you consider when starting gender-affirming hormone therapy?

Responses
A. Start estrogen and spironolactone regardless of smoking status 6% (40)
B. Start transdermal estrogen and spironolactone along with smoking cessation counseling 60% (429)
C. Do not begin hormone therapy until the patient has stopped smoking 14% (104)
D. Begin spironolactone only 20% (179)

Correct answer is B. Tobacco use combined with estrogen therapy is associated with increased venous thromboembolic risk. All transgender women who smoke should be counseled on the risks of tobacco and provided with smoking cessation options. Many transgender women may be unable or unwilling to quit smoking. Cigarette smoking, in itself, is not an absolute contraindication to starting estrogen therapy. Following an in-depth and informed consent discussion, it is reasonable to prescribe estrogen using a harm reduction approach, with a recommended route of transdermal estrogen.

To learn more about this topic, click the link below to read watch, Providing Gender-Affirming Care, presented by Jill Blumenthal, MD, MAS, at the 2022 Ryan White HIV/AIDS Program CLINICAL CONFERENCE.

A patient presents with cryptococcal meningitis, severe headache, and opening pressure greater than 25 cm H20 on day 1 of therapy. Which of the following would you initiate if the central nervous system (CNS) symptoms persist on day 2?

 

Responses
A. Dexamethasone 18% (62)
B. Acetazolamide 4% (13)
C. Mannitol 7% (24)
D. Lumbar puncture to remove fluid 53% (179)
E. Lumbar puncture for repeat diagnostic studies and magnetic resonance imaging (MRI) to assess for a second concurrent infectious or neoplastic process 18% (60)

Correct answer is D. If the CNS symptoms persist on day 2, a lumbar puncture should be administered to remove fluid.

To learn more about this topic, click the link below to read watch, Management of AIDS-Related Opportunistic Infections, presented by Henry Masur, MD, at the 2022 Ryan White HIV/AIDS Program CLINICAL CONFERENCE.

Treatment of monkeypox virus infection can include various options for therapy. To recover, most individuals will need which of the following?

Responses
A. Tecovirimat 26% (160)
B. Supportive care 61% (386)
C. JYNNEOS vaccine 8% (49)
D. Brincidofovir 5% (34)

Correct answer is B. Most individuals diagnosed with monkeypox virus infection will recover with supportive care or symptomatic management alone. Individuals who develop or who are at risk for severe disease or who are enrolled in clinical trials may receive the antiviral drug tecovirimat, if eligible.

To learn more about this topic, click the link below to read the TAM™ article, Approaching Monkeypox: A Guide for Clinicians, written by Heidi M. Torres, MD, Grant Ellsworth, MD, MS, Jason Zucker, MD, and Marshall J. Glesby, MD, PhD.

A 52-year-old woman without known HIV is diagnosed with Pneumocystis jirovecii pneumonia. Her HIV antibody test is positive, her CD4+ count is 103 cells/µL, and her HIV RNA is 135,000 copies/mL. She is still intubated on day 4 of intravenous trimethoprim-sulfamethoxazole and corticosteroids. When should she start ART?

Responses
A. Immediately 56% (432)
B. In the next 2 weeks 35% (273)
C. After completing 21 days of trimethoprim-sulfamethoxazole 8% (59)
D. At her first outpatient clinic visit 1% (12)

Correct answer is B. She should start ART in the next 2 weeks.

To learn more about this topic and other opportunistic infections, click the link below to watch, Management of AIDS-Related Opportunistic , presented by Henry Masur, MD, at the 2022 Ryan White HIV/AIDS Program CLINICAL CONFERENCE.

Which of the following diagnostic tests is most likely to have a high yield in diagnosing cryptosporidiosis?

Responses
A. Stool ova and parasite (O&P) examination 28% (207)
B. Modified acid-fast stain on a stool sample 52% (374)
C. Stool white blood cell (WBC) count 8% (55)
D. Stool culture with prolonged incubation 12% (90)

Correct answer is B. Cryptosporidium spp infection in persons with HIV who have a CD4+ count of less than 200 cells/µL often manifests as a prolonged diarrheal illness characterized by more than than 10 watery stools per day, nausea, and weight loss. It is important to note this pathogen is not accurately diagnosed using a stool O&P examination. Recommended diagnostic tests include modified acid-fast stain, immunofluorescence, and enteric polymerase chain reaction (PCR) tests.

To learn more about this topic, click the link below to watch, Top 10 Things Practitioners Are Missing in Primary Care, presented by David H. Spach, MD, at the 2022 Ryan White HIV/AIDS Program CLINICAL CONFERENCE.

Which of the following is NOT associated with developing NAFLD/NASH?

Responses
A. Weight gain associated with integrase strand transfer inhibitor use 19% (72)
B. HIV accessory protein (Vpr) 37% (147)
C. Stavudine (d4T) 10% (40)
D. Hepatitis B virus infection 34% (134)

Correct answer is D. All of the choices except hepatitis B virus are associated with the development of hepatic steatosis leading to NAFLD/NASH.

To learn more about this topic, click the link below to read the Topics in Antiviral Medicine™ article, HIV and Liver Disease: A Comprehensive Update, written by Kenneth E. Sherman, MD, PhD, and David L. Thomas, MD, MPH.

A 32-year-old man is started on his first antiretroviral regimen. Within 1 week, his serum creatinine level increased from 1.0 to 1.15 mg/dL. Which of the following drugs is most likely responsible for the 0.15 mg/dL increase in serum creatinine level?

Responses
A. Tenofovir disoproxil fumarate (TDF) 35% (216)
B. Tenofovir alafenamide (TAF) 12% (71)
C. Abacavir (ABC) 5% (92)
D. Bictegravir (BIC) 46% (283)
E. Emtricitabine (FTC) 2% (12)

Correct answer is D. Like dolutegravir (DTG), bictegravir (BIC) inhibits a proximal tubule intracellular enzyme (OCT2) that actively secretes creatinine from the bloodstream into the urine. Although only responsible for a small amount of creatinine excretion, the inhibition of OCT2 by BIC and DTG results in an almost immediate increase in serum creatinine of ~0.1 to 0.15 mg/dL. Of note, although the estimated glomerular filtration rate (eGFR) is reduced, owing to the increase in serum creatinine, the actual GFR as measured by iohexol clearance is not changed. Of note, cobicistat also increases serum creatinine by roughly the same amount (~0.1 mg/dL) due to inhibition of another proximal tubular enzyme (MATE1), resulting from the same appearance of a reduction in eGFR, as determined by Cockcroft-Gault estimation, that is not reflected in a true reduction in GFR via iohexol clearance.

To learn more about this topic, click the button below to watch HIV 101: Initiation of Antiretroviral Therapy and Primary Care for People With HIV, presented by Michael S. Saag, MD, and Melanie A. Thompson, MD, on September 28, 2022.

You are asked to consult on a newborn infant born to a parent whose last viral load in pregnancy was 200 copies/mL 3 weeks before delivery. Which of the following regimens is recommended for infant postnatal prophylaxis?

Responses
A. Zidovudine twice daily for 4 weeks 29% (240)
B. Zidovudine twice daily for 6 weeks plus lamivudine and nevirapine twice daily for 2 to 6 weeks 48% (391)
C. Abacavir plus lamivudine twice daily plus dolutegravir once daily birth for 6 weeks 11% (92)
D. Zidovudine plus lopinavir/ritonavir twice daily for 6 weeks 12% (100)

Correct answer is B. This child qualifies for presumptive treatment. Dosing and safety have been determined for zidovudine, lamivudine, and nevirapine for use in newborns and infants. A 4-week zidovudine antiretroviral (ARV) prophylaxis regimen can be used in newborns whose mothers received antiretroviral therapy during pregnancy and had a viral suppression of fewer than 50 copies/mL within 4 weeks before delivery. Abacavir is not approved by the United States Food and Drug Administration (FDA) for use in neonates and infants aged less than 1 month and is not recommended as part of a presumptive HIV therapy regimen. Dosing for dolutegravir has not been determined for infants less than 4 weeks of age. A 2-drug regimen is not recommended for presumptive therapy. Instead, a 3-drug regimen with either nevirapine or raltegravir with zidovudine and lamivudine, agents with established dosing and safety, is recommended for infants at risk for HIV acquisition.

To learn more about this topic, click the button below to watch Update on the Prevention and Treatment of Pediatric HIV Infection, presented by Elaine J. Abrams, MD, on October 6, 2022.

Your patient has been treated 3 times for bacterial STIs over the past 2 years. He is currently on cabotegravir every 2 months for PrEP. Which strategy appears to reduce his subsequent risk of contracting another bacterial STI?

Responses
A. Switching PrEP to oral TAF/FTC or TDF/FTC 6% (46)
B. Cefixime 400 mg, taken once after sex 6% (43)
C. Doxycycline 200 mg, taken once after sex 84% (622)
D. Monthly benzathine penicillin, 2.4 million units given intramuscularly 4% (33)

Correct answer is C. In the DoxyPEP (Doxycycline Postexposure Prophylaxis) study, doxycycline given as a single dose after sex reduces the incidence of gonorrhea, chlamydia, and syphilis in both people on PrEP and people with HIV. These results contrast with what was observed in a French study, which showed a reduction only for chlamydia and syphilis. The other strategies listed are either unlikely to have any effect (option A) or have not been studied (options B and D).

To learn more about this topic, click the button below to watch the Rapid Review of Data Presented at the 2022 International AIDS Conference, presented by Paul E. Sax, MD, on September 20, 2022.

Which of the following statements is true regarding current COVID-19 antiviral therapy?

Responses
A. Current oral antiviral treatments include nirmatrelvir-ritonavir (Paxlovid), molnupiravir, and remdesivir 32% (312)
B. Nirmatrelvir-ritonavir is the only preferred oral antiviral therapy in the National Institutes of Health (NIH) treatment guidelines 42% (412)
C. Nirmatelvir-ritonavir and molnupiravir are each preferred oral antiviral therapies in the NIH treatment guidelines 23% (225)
D. The Omicron BA.5 variant is resistant to all oral antivirals 3% (36)

Correct answer is B. Remdesivir is not an oral antiviral, and molnupiravir is not a preferred antiviral; it is an alternative treatment. Both available oral antivirals are active against the Omicron BA.5 variant.

To learn more about this topic, click the button below to watch Advancements in Oral Antiviral Therapy for COVID-19, presented by Jonathan Li, MD, MMSc, on September 13, 2022. Or click the button below to watch the September 15 COVID-19 Dialogue, Continued Discussion on Urgent Virus Outbreaks: COVID-19, Monkeypox, and the Resurgence of Vaccine-Preventable Diseases.

A 45-year-old woman with HIV was diagnosed in 2005 and started ART in 2012. Her CD4+ at diagnosis was 245cells/µL. She now has controlled hypertension, an HIV RNA of less than 50 copies/mL, a current CD4+ count of 560 cells/µL, and hepatitis C virus that she will soon begin treatment for. Her ACC/AHA (American College of Cardiology/American Heart Association) Pooled Cohort Equation score is 7.0%. What factor does NOT affect your decision about whether she needs a statin?

Responses
A. Current undetectable viral load 46% (325)
B. Nadir CD4+ of 245 cells/µL 19% (133)
C. Hepatitis C virus coinfection 18% (129)
D. Prolonged viremia in the past 17% (121)

Correct answer is A.  A currently undetectable viral load does not compensate for a history of prolonged uncontrolled viremia, which, like B, and C, are HIV risk-enhancing factors that increase the risk of cardiovascular disease.

To learn more about this topic, click the link below to watch “Aging and HIV: If I knew I’d Live This Long I Would Have Moisturized,” presented by Melanie A. Thompson, MD, on September 8 in Los Angeles, California.

M.S. is a 70-year-old man diagnosed with HIV 30 years ago. He has been on efavirenz (EFV)/lamivudine (3TC)/tenofovir disoproxil fumarat (TDF) for the past 10 years. Recalling how he struggled through multiple regimens in the past, he has been very reluctant to change a regimen that “saved his life.” However, he is now willing to do so due to persistent insomnia and depressive disorder. His CD4+ count is 700 cells/µL, and his viral load is less than 20 copies/mL. He has multiple comorbidities, including type 2 diabetes, cardiovascular disease, chronic obstructive pulmonary disease, and osteoporosis. Contrary to his HIV disease, these comorbidities are poorly controlled. Which of the following statements are true?

Responses
A. The burden of comorbidities is increasing among aging people with HIV (PWH). Compared to 2005-2009, the cumulative incidence of myocardial infarction among PWH has declined and is no longer higher than compared to the uninfected population 15% (108)
B. Non-AIDS complications are largely due to antiretrovirals. Patients with spontaneous control of HIV (without ART) have a lower risk of non-AIDS events compared to PWH on suppressive ART 4% (28)
C. The life expectancy of PWH has significantly improved in the modern ART era and is now similar to that of the general population 51% (362)
D. Among PWH, age greater than 65 years is a significant risk factor for both non-communicable diseases and in-hospital mortality from COVID-19
30% (211)

Correct answer is D. The burden of comorbidities is expected to continue to increase significantly in the aging population with HIV. While life expectancy has significantly improved, there remains a survival gap among PWH compared to the general population. A recent analysis has suggested that the cumulative incidence of myocardial infarctions is increasing in PWH over the past decade compared to the general population. Among PWH, age greater than 65 years, low CD4+ cell count, chronic kidney disease, and diabetes mellitus have been shown to be factors of increased in-hospital mortality with COVID-19.

To learn more about comorbidities in older people with HIV, click the link below to watch the on-demand recording, “In Case You Missed It: The latest in HIV Literature and Monkeypox,” presented by Roger J. Bedimo, MD, MS, at the September 8 course in Los Angeles.

You have a patient who presents with Human Monkeypox Virus, which has been confirmed by assay. Which of the following medications is recommended as the first-line treatment for this patient during the 2022 outbreak?

 

Responses
A. Acyclovir 5% (37)
B. Cidofovir 4% (33)
C. Brincidofovir 5% (36)
D. Tecovirimat 77% (608)
E. Vaccinia immunoglobulin 9% (72)

Correct answer is D. Tecovirimat is recommended as the first-line treatment during this 2022 Human Monkeypox Virus outbreak. However, there is limited safety data and no human efficacy data to date. Acyclovir is not recommended due to its need for phosphorylation by viral kinases and is much more effective against herpes virus infections than poxvirus infections. Cidofovir is not recommended because, although it is likely to have activity against orthopoxviruses, concerns about nephrotoxicity limit generalized use. Brincidofovir is not recommended because, although it is likely to have activity against orthopoxviruses, concerns about hepatotoxicity limit generalized use. Vaccinia immunoglobulin is not recommended because it has no proven benefit, but it may be used for postexposure prophylaxis in individuals who cannot receive the vaccine.

To learn more information about Human Monkeypox Virus, click the button below to watch the August 30, 2022, webinar, What You Need to Know About the Diagnosis and Treatment of Human Monkeypox Virus, presented by Jason E. Zucker, MD.

Which of the following ways does long-acting early viral inhibition (LEVI) differs from acute HIV infection (AHI)?

 

Responses
A. LEVI is often asymptomatic or protean 49% (517)
B. LEVI is briefer than AHI 10% (101)
C. LEVI is often associated with acute opportunistic infections 10% (109)
D. LEVI is associated with HIV acquisition within 12 months of a long-acting cabotegravir injection 31% (330)

Correct answer is A. LEVI is often asymptomatic or protean.

To learn more about this topic, click the button below to register for the 30th Annual Update on HIV Management in Los Angeles.

TRUE or FALSE: In a study by Diggins and colleagues, the primary hypothesis was that teduglutide may improve immune activation and downstream arterial inflammation in people with HIV by preserving the intestinal epithelial barrier.

 

Responses
A. True 84% (622)
B. False 16% (117)

Correct answer is A. Disruption of the intestinal epithelial barrier is seen in people with HIV, and the proof-of-concept study by Diggins and colleagues was designed to determine whether teduglutide, a GLP-2 agonist, may have a beneficial effect on the intestinal epithelial barrier in people with HIV, thereby by reducing immune activation and downstream arterial inflammation.

To learn more about this topic, click the button below to read the TAMTM article, CROI 2022: Metabolic and Other Complications of HIV Infection or COVID-19, written by Sudipa Sarkar, MD, and Todd T. Brown, MD, PhD.

Which of the following findings was reported in the Tsepamo study, in which birth outcome surveillance was conducted among women in Botswana who were routinely screened for COVID-19 at delivery?

Responses
A. Maternal mortality was higher, with an age-adjusted risk ratio of 31.6, in women with COVID-19 than in women without COVID-19 47% (142)
B. Maternal mortality was higher during pre-Delta COVID-19 variant waves than during the Delta wave 10% (29)
C. Rates of any adverse birth outcome (defined as preterm delivery, small for gestational age, stillbirth, and neonatal death) were similar among infants born to women with COVID-19 and women without COVID-19 20% (62)
D. Rates of any adverse birth outcome were similar among infants born to women with HIV and who had COVID-19 and among infants born to women in the other comparison groups 23% (70)

Correct answer is A. In the Tsepamo study, which conducted birth outcomes surveillance among women in Botswana, maternal mortality was higher, with an age-adjusted risk ratio of 31.6, in women with COVID-19 than in women without COVID-19. Maternal mortality was higher during the wave of the Delta COVID-19 variant than during pre-Delta waves. Rates of any adverse birth outcome were significantly higher among infants born to women with COVID-19 than among women without COVID-19. Rates of any adverse birth outcome were highest among infants born to women with HIV and who also had COVID-19.

For more information, click the link below to read the TAM article, CROI 2022: Advances in Antiviral Therapy for HIV, COVID-19, and Viral Hepatitis, written by Shauna H. Gunaratne, MD, MPH, Hong-Van Tieu, MD, MS, Timothy J. Wilkin, MD, MPH, and Barbara S. Taylor, MD, MS.

Which of the following groups would be eligible for simplified treatment regimens for hepatitis C virus (HCV)?

Responses
A. Persons with HIV 36% (278)
B. Pregnant persons 9% (73)
C. Patients with compensated cirrhosis 40% (308)
D. Patients with a positive hepatitis B surface antigen 15% (112)

Correct answer is C. Direct-acting antiviral regimens are available for people with HCV, whether they have no evidence of cirrhosis or compensated cirrhosis. HIV coinfection requires attention to potential drug interactions. Data for the treatment of pregnant persons are few. If hepatitis B virus (HBV) coinfection is present, there is a need for additional monitoring due to the risk of HBV reactivation.

To learn more about hepatitis C virus treatment, click the button below to watch the on-demand webinar, Simplifying Treatment of Hepatitis C and Overcoming Barriers to Cure, presented by Arthur Y. Kim, MD, on August 2, 2022.

Drug interactions between rifampin and integrase strand transfer inhibitors occur because rifampin is which of the following?

Responses
A. Uridine glucuronosyltransferases (UGT1A) substrate 6% (49)
B. Cytochrome P450 3A (CYP3A) inhibitor 47% (362)
C. UGT1A inducer 48% (294)
D. P-glycoprotein (p-gp) inhibitor 9% (68)

Correct answer is C. Rifampin is a potent inducer of CYP3A4, UGT1A, and p-gp. Since bictegravir is a substrate of CYP3A, UGT1A1, and p-gp, rifampin-induced induction will reduce bictegravir levels leading to a loss of virologic suppression and risk of resistance. Rifampin does not inhibit CYP3A or p-gp.

To learn more about the drug-drug interactions of antiretroviral therapy and tuberculosis, click the button below to watch the on-demand webinar, Key Antiretroviral Drug-Drug Interactions With Mycobacterium tuberculosis Infection in Persons With HIV: A Case-Based Presentation, presented by Betty J. Dong, PharmD, on July 26, 2022.

Which of the following groups should be considered for a 3-dose primary SARS-CoV-2 vaccination series rather than the standard 2-dose series?

Responses
A. Current CD4+ count greater than 350 cells/µL and older than 65 years 7% (39)
B. Current CD4+ count less than 500 cells/µL and older than 65 years 7% (40)
C. Current CD4+ count less than 200 cells/µL or unsuppressed HIV viral load 48% (280)
D. Current CD4+ count less than 200 cells/µL and unsuppressed HIV viral load 17% (103)
E. Current CD4+ count less than 200 cells/µL and older than 65 years 21% (127)

Correct answer is C. For PWH who have a current CD4+ count of less than 200 cells/µL or have a detectable HIV viral load, a 3-dose primary SARS-CoV-2 vaccination series should be considered.

To have more of your difficult COVID-19 questions answered, click the below button to watch the on-demand course, COVID-19 Prevention and Management: HIV Coinfection, Outpatient Management, Hospitalization, and Post-COVID-19 Complications.

Which of the following initial HIV treatment regimens containing hepatitis B virus (HBV) active agents is not appropriate for HBV therapy in people with HIV?

 

Responses
A. Bictegravir (BIC)/emtricitabine (FTC)/tenofovir alafenamide (TAF) 8% (69)
B. Dolutegravir (DTG) plus TAF/FTC 4% (39)
C. DTG/abacavir (ABC)/lamivudine (3TC) 59% (540)
D. DTG plus tenofovir disoproxil fumarate (TDF)/3TC 7% (66)
E. All are acceptable regimens to treat both HBV and HIV 22% (206)

Correct answer is C. Although 3TC and FTC each some have activity against HBV, using either these anti-HBV agents alone (monotherapy ) for HBV infection is considered suboptimal due to high rates of resistance emergence over time and less potent HBV DNA suppression compared with TAF, TDF, or entecavir (ETV) regimens. HIV regimens for someone with HBV coinfection should include TAF or TDF, or include ETV if TAF or TDF cannot be used.

For more information about HIV and HBV co-infection, click the link below to watch the the July 19 webinar, Unique Issues in People With HIV and Hepatitis B/D Coinfections, presented by David L. Wyles, MD.

Which of the following recommended therapies has the least clinical evidence to support its use for high-risk outpatients with COVID-19?

Responses
A. Bebtelovimab 45% (348)
B. Molnupiravir 22% (167)
C. Nirmatrelvir-ritonavir 12% (93)
D. Remdesivir 21% (163)

Correct answer is A. Recommendation for its use is based on in vitro activity against current omicron variants; clinical data are very limited, and thus bebtelovimab has not been shown to reduce hospitalizations in a randomized controlled trial. Remdesivir, nirmatrelvir-ritonavir, and molnupiravir were each shown to reduce hospitalizations in placebo-controlled randomized trials.

To learn more about COVID-19, click the button below to watch the latest COVID-19 Dialogue from July 7, 2022. For an in-depth review of in-patient and outpatient care, click the button below to register for the virtual CME activity, COVID-19 Prevention and Management: HIV Coinfection, Outpatient Management, Hospitalization, and Post-COVID-19 Complications. This CME activity is scheduled for July 20, 2022.

Which of the following PrEP regimens are recommended for coverage for a missed long-acting injectable cabotegravir visit?

 

Responses
A. Daily oral tenofovir disoproxil fumarate/emtricitabine (TDF/FTC) 37% (178)
B. Daily oral tenofovir alafenamide/emtricitabine (TAF/FTC) 14% (70)
C. Daily oral cabotegravir (CAB) 38% (186)
D. On-demand tenofovir disoproxil fumarate/emtricitabine (TDF/FTC) 11% (54)

Correct answer is A. The package insert for cabotegravir recommends oral cabotegravir for up to 2 months after a missed injection visit. After 2 months, an alternative form of PrEP should be used. Oral cabotegravir has not been studied as an oral PrEP option. Therefore, based on the data for daily oral TDF/FTC, as well as cost and implementation considerations, TDF/FTC would be the best option for covering missed injection visits.

To learn more about long-acting injectable CAB, click the link below to watch Implementation of Long-Acting PrEP: Case-Based Panel Discussion With Clinicians, presented by Colleen Kelley, MD, MPH, Kim Scarsi, PharmD, and Hyman Scott, MD, MPH, on June 28, 2022.

Which of the following statements regarding the pharmacokinetic (PK) and PK-pharmacodynamics (PD) of nirmatrelvir/ritonavir is CORRECT?

 

Responses
A. Nirmatrelvir is highly protein bound 12% (62)
B. The protein-adjusted 90% effective concentration (EC90) for nirmatrelvir is 292 ng/mL 28% (139)
C. Race, weight, and renal function influence nirmatrelvir PK 23% (112)
D. In the EPIC-SR (Evaluation of Protease Inhibition for COVID-19 in Standard-Risk Patients) and EPIC-HR (Evaluation of Protease Inhibition for COVID-19 in High-Risk Patients) trials in persons with COVID-19, those with a nirmatrelvir trough above 1000 ng/mL had faster time to symptom resolution 37% (186)

Correct answer is B.  The protein-adjusted EC90 for nirmatrelvir is 292 ng/mL. Option A is incorrect because nirmatrelvir is 69% bound. Option C is incorrect because no population PK studies have been reported to date that evaluate clinical or demographic factors associated with nirmatrelvir PK. Option D is incorrect because no PK-PD associations have been reported with nirmatrelvir in persons with COVID-19 to date.

To learn more about the PK-PD of antivirals, click the button below to watch the on-demand webinar Optimizing Antiviral Outcomes Through Clinical Pharmacology, presented by Jennifer J. Kiser, PharmD, PhD, on June 21, 2022.

Which of the following antiretroviral therapy (ART) regimens is now recommended for all pregnancy-related clinical categories for people with HIV (PWH)?

 

Responses
A. Bictegravir/tenofovir
alafenamide/emtricitabine
18% (139)
B. Doravirine/tenofovir disoproxil
fumarate/lamivudine
8% (60)
C. Dolutegravir/tenofovir
alafenamide/emtricitabine
60% (456)
D. Cabotegravir/rilpivirine 3% (27)
E. Dolutegravir/lamivudine 11% (84)

Correct answer is C. Dolutegravir/tenofovir alafenamide/emtricitabine is the preferred regimen for people who are pregnant or trying to become pregnant.

To learn more about ART during pregnancy, along with other clinical situations, click the button below to watch, Challenging Cases in Antiretroviral Therapy and Panel Discussion, presented by Paul E. Sax, MD, at the IAS–USA 30th Annual Scott M. Hammer Update on HIV Management on June 13, 2022, in New York, NY.

Which of the following statements is true regarding comorbidities among adolescents with HIV?

 

Responses
A. Adolescents with HIV have lower rates of mental health disorders than adolescents without HIV 2% (13)
B. Untreated mental health disorders result in nonadherence and increased risk of morbidity 91% (538)
C. Adolescents do not experience the same weight gain with integrase strand transfer inhibitors (InSTIs) that adults do 3% (16)
D. Adolescents and young adults with HIV are less likely to be sexually active than those without HIV 2% (12)
E. Human papillomavirus (HPV) vaccination cannot be given to adolescents with HIV 2% (10)

Correct answer is B. There is a known association between unaddressed mental health and nonadherence and poor outcomes.

To learn more about HIV-related comorbidities in adolescents, click the button below to read the TAM™ article, Preventing and Diagnosing HIV-Related Comorbidities in Adolescents, written by Hasiya Eihuri Yusuf, MD, MPH; David Griffith, MD; and Allison Lorna Agwu, MD, ScM.

Which antiretroviral medication interacts with metformin so that the dose of metformin should not exceed 1000 mg daily with concomitant use?

 

Responses
A. Doravirine 20% (266)
B. Dolutegravir 68% (926)
C. Start ART at end of TB treatment (6 months) 4% (49)
D. Raltegravir 8% (116)

Correct answer is B. Coadministration of dolutegravir and metformin increases metformin concentrations by about 80%. As a result, the maximum daily dose of metformin in the setting of dolutegravir should be 1000 mg daily.

To learn more about managing diabetes in people with HIV, click the button below to watch the May 31, 2022, webinar, Management of Diabetes in People With HIV, presented by Todd T. Brown, MD, PhD.

A 54-year-old woman is admitted to your hospital with a cough, a fever, and weight loss. She is diagnosed with HIV on admission and is found to have a CD4+ count of 70 cells/μL and her HIV RNA is 120,000 copies/mL. Chest X-ray shows pleural thickening, and diffuse infiltrates. A bronchoscopy is performed, and the smear microscopy is negative for acid-fast bacilli. Gene Xpert Mycobacterium tuberculosis (Mtb)/rifampicin subsequently confirms tuberculosis (TB); the sputum culture is pending. She is started on treatment for TB with isoniazid, rifampin, ethambutol, and pyrazinamide. When should you start her on antiretroviral therapy (ART)?

Responses
A. Start ART as soon as possible (within 2 weeks) 59% (389)
B. Start ART within 8 weeks 37% (247)
C. Start ART at end of TB treatment (6 months) 4% (25)

Correct answer is B. Based on a number of clinical trials (CAMELIA [Early vs Late Introduction of Antiretroviral Therapy in HIV-infected Patients With Tuberculosis], SAPIT [Starting Antiretroviral Therapy at Three Points in Tuberculosis], STRIDE [Strategies to Reduce Injuries and Develop Confidence in Elders]), there is now clear evidence that early ART initiation is safe and associated with reduced mortality. There is no increased risk of adverse events; however, earlier ART initiation is associated with an increase in the risk of immune reconstitution inflammatory syndrome (IRIS). Therefore, the American Thoracic Society (ATS), the US Department of Health and Human Services (DHHS), and the International Antiviral Society–USA (IAS–USA) all recommend early ART initiation in patients with TB (early is defined as within 2 weeks for those with a CD4+ count below 50 cells/μL) and within 8 weeks for those with a CD4+ count greater than 50 cells/μL. The exception to early initiation is in cases of TB meningitis, where increased rates of adverse events and death have been reported with early ART initiation. The correct answer for this case, in which the patient has a CD4+ count of 70 cells/μL, is option B.

To learn more about treating HIV and tuberculosis, follow the link below to read the new Topics in Antiviral Medicine™ article, Update on Tuberculosis/HIV Coinfections: Across the Spectrum From Latent Infection Through Drug- Susceptible and Drug-Resistant Disease, written by Elisa H. Ignatius, MD, MSc, and Susan Swindells, MBBS.

A 38-year-old man diagnosed with HIV comes to the office expressing passive suicidal ideation. During the past 6 weeks, he has experienced fatigue, loss of interest in usual activities, a 7-kg (15-lb) weight loss, and insomnia. He is diagnosed with a major depressive episode. Which of the following is NOT true regarding his depression treatment?

Responses
A. Beginning a low dose of antidepressant reduces the risk of adverse effects 9% (53)
B. Escalating the dose of an antidepressant medication should be avoided 54% (304)
C. Accepting treatment with antidepressants is associated with greater adherence to antiretroviral medications 21% (117)
D. More than one antidepressant trial may be needed to achieve remission from depression 16% (87)

Correct answer is B. If the patient can tolerate the adverse effects of an antidepressant, it may be necessary to gradually increase the dose to achieve a therapeutic response.

For more information on treating depression in people with HIV, follow the link below to read the new Topics in Antiviral Medicine™ article, Addressing Depressive Disorders Among People With HIV, written by Andres Fuenmayor, MD, and Francine Cournos, MD.

Long-acting (LA) therapy with cabotegravir plus rilpivirine (CAB/RPV) as a switch therapy in adults with HIV suppressed on therapy has been studied in 3 large randomized controlled trials. Which of the following is a common result across these studies?

Responses
A. LA CAB/RPV was superior to continued oral therapy in maintaining suppression of HIV RNA 16% (118)
B. Virologic failure occurred in less than 3% of participants treated with LA CAB/RPV in each study 54% (395)
C. When virologic failure occurred, only nonnucleoside reverse transcriptase inhibitor (NNRTI) resistance mutations emerged 8% (56)
D. LA CAB/RPV therapy was always preceded by oral lead-in therapy 20% (147)
E. Injection site reactions increased in severity over time 2% (19)

Correct answer is B. Across all studies (FLAIR [First Long-Acting Injectable Regimen] and ATLAS [Antiretroviral Therapy as Long Acting Suppression]), LA CAB/RPV was noninferior to continued oral therapy, virologic failure, and an HIV viral load of greater than 50 copies/mL occurred 1% to 3% of the time depending on the study. When resistance occurs, there is typically resistance to NNRTIs and to integrase strand transfer inhibitors (InSTIs). In FLAIR crossover from oral therapy at week 96, participants were randomized to CAB/RPV oral lead or to direct to intramuscular (IM) injection of LA CAB/RPV. Finally, in each study, injection site reaction severity decreased over time.

To learn more about LA CAB for treatment, click the button below to watch NEW ANTIRETROVIRALS AND THE FUTURE OF HIV TREATMENT AND PREVENTION presented by Chloe M. Orkin, MBBCH, MSC, from Queen Mary University of London on February 16, 2022, at CROI 2022.

Nirmatrelvir/ritonavir has United States Food and Drug Administration (FDA) Emergency Use Authorization to treat mild-to-moderate COVID-19 in patients at high risk of progression to severe disease, who are 12 years of age and older, and weigh 40 kg or more. In such patients, nirmatrelvir/ritonavir should be initiated within 5 days of symptom onset. Given the coformulation with ritonavir, there is some risk of drug interactions. Which of the following treatments is absolutely contraindicated (ie, cannot be temporarily discontinued or the dose adjusted) while taking nirmatrelvir/ritonavir?

Responses
A. Atorvastatin 23% (133)
B. Bictegravir/emtricitabine/tenofovir alafenamide 21% (127)
C. Salmeterol 33% (199)
D. Bupropion 6% (38)
E. Risperidone
17% (179)

Correct answer is C. Coadministration of salmetrol and nirmatrelvir/ritonavir may increase cardiac effects, and nirmatrelvir/ritonavir should be avoided. Nirmatrelvir/ritonavir may increase bictegravir drug levels, but this has no clinical significance, and no dose modification is needed. Nirmatrelvir/ritonavir increases the levels of most statins. Atorvastatin and all other statins can be withheld during nirmatrelvir/ritonavir treatment and for 5 days thereafter. Nirmatrelvir/ritonavir increases drug levels of risperidone. No dose adjustment is recommended, but monitoring for adverse events is necessary.

Sources: IDSA, Liverpool, and the IAS–USA May 6 COVID-19 Dialogue

To learn more about nirmatrelvir/ritonavir, click the button below to watch the May 6, 2022, IAS–USA COVID-19 Dialogue titled, Detailed Discussion on the Most Current Issues in COVID-19 Therapeutics, 2nd Booster, Variants of Variants, and COVID-19 in Children, moderated by Dr Paul A. Volberding with discussants Drs Yvonne Maldonado, Peter Chin-Hong, and Carlos del Rio.

In a male patient who has treated HIV and primary syphilis, which one of the following is recommended by the Centers for Disease Control and Prevention (CDC) sexually transmitted infections (STIs) guidelines panel?

Responses
A. Lumbar puncture to evaluate for neurosyphilis 5% (29)
B. Repeat non-specific syphilis serology (ie, rapid plasma reagin [RPR]) at 6, 9, 12, and 24 months after treatment 48% (270)
C. Penicillin G benzathine 2.4 million units (MU) intramuscularly (IM) weekly for 3 weeks 15% (81)
D. Thorough ocular and otic examination 32% (179)

Correct answer is D. The CDC sexually transmitted infections guidelines panel recommends that all persons with HIV infection and primary and secondary syphilis have a thorough neurologic, ocular, and otic examination.

For more information on screening for and managing STIs, click the button below to watch “Controversies in Managing Sexually Transmitted Infections: A Case-Based Panel Discussion,” presented by Jeffrey L. Lennox, MD, at the 30th Annual Update on HIV Management course in Atlanta, Georgia.

Which of the following is true regarding the incidence rate of anal cancer among persons with HIV?

Responses
A. They are low among women with HIV 5% (40)
B. They are approximate to the rates of common cancers that screening guidelines exist for 27% (192)
C. They continue to rise despite the widespread use of ART 52% (378)
D. They are only high in those with a history of anoreceptive sex 16% (114)

Correct answer is B. Anal cancer rates for women with HIV approximate the historic cervical cancer rates prior to widespread cervical cancer screening. Among men who have sex with men (MSM) with HIV, the rates approximate or exceed the rates of lung, colon, and prostate cancer.

For more information on screening for anal cancer, click the button below to watch “Screening for Anal Cancer: When to Screen and What to Do With the Results,” presented by Grant Ellsworth, MD, at the 30th Annual Update on HIV Management course in Atlanta, Georgia.

Which of the following is NOT true?

Responses
A. Gait speed alone predicts disability and death 25% (112)
B. Frailty predicts risk for diabetes 17% (78)
C. American College of Cardiology (ACC) and the American Heart Association (AHA) cardiovascular risk of greater than or equal to 7.5% is associated with an increased risk of frailty 15% (69)
D. A short Physical Performance Battery score of less than 10 predicts a higher risk of death 14% (63)
E. The Clinical Frailty Score requires a timed walk 29% (129)

Correct answer is E. The clinical frailty scale is a quick “eyeball” test of frailty that is completed in less than a minute and does not include a timed walk.

To learn more about assessing frailty in older adults with HIV, click the button below to watch “Frailty: Screening, Preventing, and Intervening,” presented by Melanie Thompson, MD, at the 30th Annual Update on HIV Management in Atlanta, Georgia.

Which of the following statements about long-acting cabotegravir (CAB-LA) is FALSE?

Responses
A. In the first year of unblinded data, it was still shown to be 66% more effective than tenofovir disoproxil fumarate (TDF)/emtricitabine (FTC) at preventing HIV 14% (91)
B. There have been no cases of breakthrough infections in the participants who received injections on time 49% (318)
C. Some people with breakthrough infections developed integrase strand transfer inhibitor (InSTI) resistance 20% (112)
D. RNA testing is recommended at baseline and every 2 months 17% (81)

Correct answer is B. There have been 7 reported cases of breakthrough infections in participants who received injections on time.

To learn more about CAB-LA and other preventative strategies, click the button below to watch “CROI 2022 Update: Highlights in HIV and COVID-19 Epidemiology and Prevention​,” presented on March 31, 2022, by Susan P. Buchbinder, MD.

In people who inject drugs (PWID), PrEP uptake falls in the range of:

Responses
A. 0%-3% 42% (373)
B. 5%-10% 28% (252)
C. 10%-15% 20% (178)
D. 20%-25% 10% (81)

Correct answer is A. Despite high awareness and willingness, PrEP uptake has been extremely low in PWID (0%-3%). Declines in engagement are observed through the PrEP care continuum, and there is a paucity of data on sexual minority men who inject drugs.

For more information on HIV prevention in special populations, click the button below to watch the HIV PREVENTION IN SPECIAL POPULATIONS symposium presented on February 24, 2022, at CROI 2022.

What is the recommended timing for hepatitis C virus (HCV) treatment initiation in the setting of acute/recent infection or documented reinfection? .

Responses
A. After waiting 3 to 6 months to evaluate for spontaneous clearance 26% (179)
B. Immediately or as soon as feasible 52% (362)
C. Only once chronic infection is established (more than 6 months) 11% (73)
D. After 1 month to ensure acute hepatic failure does not develop 4% (25)
E. Once resistance testing results are back 7% (51)

Correct answer is B. The major guidelines and guidance documents recommend expedited or immediate treatment of acute HCV infection without waiting for spontaneous clearance assessment. This also applies to the setting of documented HCV reinfection. This recommendation is rooted in the public health benefit with prevention of onward transmission as well as the benefit to the individual. Baseline resistance testing is not routinely recommended.

To learn more about liver disease and hepatitis B, C, and D, click the button below to watch the CASE-BASED LIVER WORKSHOP presented on February 13, 2022, at CROI 2022.

True or false: The new PrEP Guidelines from the Centers for Disease Control and Prevention (CDC) recommend all sexually active adults and adolescents should receive information about PrEP.

Responses
A. True 93% (1225)
B. False 7% (86)

Correct answer is A. The new recommendation for clinicians is to inform all sexually active adults and adolescents about PrEP, and providers to offer PrEP to anyone who requests it, even if they do not report specific risk behaviors. This recommendation is intended to increase awareness and make PrEP available to people who may be apprehensive about sharing potentially stigmatizing HIV risk behaviors with their clinician.

To learn more about the new PrEP guidelines, be sure to watch the March 22, 2022, webinar, CDC PrEP Clinical Practice Guideline and Strategies for Ending the HIV Epidemic in the US, presented by Demetre C. Daskalakis, MD, MPH, and Dawn K. Smith, MD, MS, MPH.

At CROI 2022, the 3rd case of HIV cure was reported in a woman with well-controlled HIV who developed acute myeloid leukemia (AML). Which of the following is TRUE?

Responses
A. The participant was of mixed race and lived in a developing country 6% (43)
B. The participant received systemic chemotherapy, but NOT total body irradiation 8% (57)
C. The participant received a bone marrow transplant from a donor with the delta 32 deletion in the gene coding for the CCR5 recepto 52% (387)
D. The participant never experienced graft vs host disease
34% (248)

Correct answer is D. The participant never experienced graft vs host disease. Like the first 2 documented cases of HIV cure, this participant, a woman of mixed race living in New York, was living with well-controlled HIV when she developed aggressive AML. She enrolled in the IMPAACT P1007 study and following systemic chemotherapy (but NOT total body irradiation) for her AML, she received an umbilical cord blood transplant that was pre-screened and harbored the delta 32 deletion. She engrafted the cord blood cells but NEVER developed graft vs host disease, which was hypothesized to play a role in the first 2 documented cases of HIV cure. Following discontinuation of antiretroviral therapy for more than a year, she remains HIV-free and is considered cured.

To learn more about this cure patient, as well as other CROI 2022 updates, be sure to watch the March 15, 2022, on-demand webinar, CROI 2022 Update: Highlights in Clinical Research With a Focus on Antivirals, presented by Roy M. Gulick, MD, MPH.

As a reminder, CROI 2022 content will become public on the CROI website on March 25, 2022.

AB is a 32-year-old Black woman who has been virologically suppressed on efavirenz (EFV)/tenofovir disoproxil fumarate (TDF)/emtricitabine (FTC) since 2012. She developed progressively worsening insomnia and was diagnosed with depressive disorder. Her antiretroviral therapy (ART) regimen was switched to abacavir (ABC)/lamivudine (3TC)/dolutegravir (DTG) 2 years ago. She has since then gained 25 lbs. She is not concerned about her appearance, but you would like to counsel her about potential metabolic complications of weight gain on ART. Which of the following are metabolic complications of weight gain on ART?

Responses
A. DTG is associated with significant worsening of lipid profile with correlate with the weight gain 9% (55)
B. There is no risk for metabolic complications. Most of the weight gain is lean, not fat mass 4% (24)
C. Weight gain on ART is associated with an increased risk of cardiovascular disease (CVD) 14% (86)
D. Weight gain on ART is associated with an increased risk of metabolic syndrome 73% (438)

Correct answer is D. Previous studies have shown that obesity disproportionately increases diabetes risk in people with HIV compared to HIV-uninfected controls. In the D:A:D (Data Collection on Adverse Events of Anti-HIV Drugs) study cohort, a higher body mass index (BMI) was not associated with an increased risk of CVD, but such changes were consistently associated with an increased risk of diabetes mellitus (DM). Weight gain was also associated with an increased risk of metabolic syndrome in the ADVANCE (Action in Diabetes and Vascular Disease: Preterax and Diamicron Modified Release Controlled Evaluation) trial. DTG is not associated with an increased risk of dyslipidemia.

To learn more about weight gain and ART, click the Watch Webcast button below to view the our March 8th webinar on Weight Gain: A Growing Issue in Antiretroviral Therapy, presented by Roger J. Bedimo, MD, MS.

By what percentage did new HIV infections decline in the United States from 2015 to 2019?

Responses
A. No decline 13% (128)
B. 8% 44% (443)
C. 16% 22% (227)
D. 24% 16% (162)
E. 32% 5% (49)

Correct answer is B. There was only an 8% decline in the United States from 2015 to 2019, despite scale-up of testing and treatment. Preexposure prophylaxis (PrEP) scale-up could have a substantial impact on reducing new infections nationally, and getting us toward the Ending the HIV Epidemic goal of a 75% reduction in new infections by 2025, and a 90% reduction in new infections by 2030.

To learn more about how PrEP can be utilized in meeting the Ending the HIV Epidemic goals, click the Watch Webcast button below to view the presentation, “Ending the HIV Epidemic: The Impact of PrEP,” presented by Susan P. Buchbinder, MD, at the November 19, 2021, virtual course, PrEParing for PrEP: From Policy to Implementation.

Which of the following statements is FALSE?

 

Responses
A. Treating depression may require considerable trial and error 11% (108)
B. Mood stabilizers tend to have more interactions with antiretroviral medications than antidepressants 16% (146)
C. More than 10% of patients diagnosed with major depression in a primary care setting have bipolar depression 16% (145)
D. Antidepressants are a good first-line treatment for bipolar depression 44% (405)
E. Ketamine is now being used for refractory depression 13% (127)

Correct answer is D. Antidepressants are not the recommended treatment for bipolar depression and can precipitate mania.

For more information, click the Watch Webcast button below to view the presentation, Addressing Depression in Patients with HIV, presented by Francine Cournos, MD, at the virtual 2021 Ryan White HIV/AIDS Program CLINICAL CONFERENCE on October 5, 2021.

Chronic pain can be influenced by a variety of diseases and environmental factors. Which of the following is NOT indicative of the potential worsening of chronic pain?

Responses
A. Frequent psychiatric follow-up visits 39% (136)
B. Inadequate housing 28% (98)
C. A high HIV viral load 30% (106)
D. Insomnia 3% (12)

Correct answer is A. Undertreated mental health conditions can worsen chronic pain. However, having regular, frequent psychiatric follow-ups can improve the care of mental health conditions and help reduce chronic pain. Inadequate housing and insomnia can lead to increased physical and psychologic stress levels, and a high viral load may contribute to HIV-induced nerve damage. These latter factors can lead to a worsening in chronic pain.

For more information on chronic pain and HIV, follow the link below to read the new Topics in Antiviral Medicine™ article, Chronic Pain and Opioid Use in Older People With HIV, written by Vasudev C. Mandyam, MD, and R. Douglas Bruce, MD, MA, MS.

Which of the following is/are the most commonly identified risk factor(s) of cognitive decline in people with HIV on suppressive antiretroviral therapy (ART)?

 

Responses
A. A history of HIV-associated neurocognitive disorder (HAND) or cognitive impairment pre-ART, and older age
83% (679)
B. Asymptomatic cerebral spinal fluid (CSF) viral escape 4% (34)
C. Reversed CD4+/CD8+ cell ratio 5% (37)
D. Low central nervous system penetration effectiveness (CPE) ART regimens 8% (66)

Correct answer is A. Longitudinal studies and a meta-analysis suggest that older people with HIV and those with a pre-existing cognitive impairment before ART had a higher risk of cognitive decline than those without these risk factors. Although CSF samples in asymptomatic CSF viral escape demonstrated higher levels of immune activation markers than those without escape, it remains unclear if the former poses a substantial negative impact on long-term cognitive function. Reversed CD4+/CD8+ ratio is common among people with HIV who are virally suppressed and is associated with noncommunicable comorbidities, although its association with long-term cognitive function is unclear. The use of low CPE ART regimens may be associated with a higher risk of CSF viral escape. However, low CPE ART regimens appear safe in uncomplicated cases and are not associated with cognitive decline.

For more information on cognition and HIV, follow the link below to read the new Topics in Antiviral Medicine™ article, Neurocognition and the Aging Brain in People With HIV: Implications for Screening, written by Philip Chan, MBChB, PhD, and Victor Valcour, MD, PhD.

Long-term opioid treatment (LTOT) for chronic non-malignant pain in people with HIV increases retention in HIV treatment by what percent?

 

Responses
A. Does not increase retention 34% (249)
B. Increases retention by 10% 11% (76)
C. Increases retention by 15% 15% (105)
D. Increases retention by 20% 21% (153)
E. Increases retention by 25% 19% (140)

Correct answer is A. Although patients with chronic pain and not on LTOT had twice the rate of virologic failure than those on LTOT, LTOT did not impact retention in HIV treatment in patients with pain.

For more information on opioid use, follow the link below to view the presentation, “Chronic Pain and Opioid Use in Older People with HIV,” presented by R. Douglas Bruce, MD, MA, MS, at the June 25, 2021, virtual course, Aging and HIV: Issues, Screening, and Management in Individuals with HIV as They Age.

This topic will also be covered in the next issue of Topics in Antiviral Medicine. This article will be released soon; check the IAS–USA website for updates.

Which of the following is a Centers for Disease Control and Prevention (CDC) preferred treatment approach for latent tuberculosis (TB) infection?

Responses
A. Isoniazid daily for 9 months 32% (205)
B. Rifampin and isoniazid weekly for 3 months 51% (325)
C. Close monitoring of the patient’s health status and treatment only if TB disease develops 9% (60)
D. Rifampin and pyrazinamide for 6 months 8% (54)

Correct answer is B. Rifampin and isoniazid weekly for 3 months is the CDC’s preferred approach for treating latent TB. Isoniazid on its own is no longer a preferred regimen because the CDC only lists shorter course regimens. Watchful waiting is not recommended. Rifampin with pyrazinamide is not on the CDC’s list and was associated with increased mortality.

For more information, click the Watch Webinar button below to view the presentation, Update on Tuberculosis/HIV Coinfection: Across the Spectrum From Latent Infection Through Drug-Susceptible and Drug-Resistant Disease, presented by Susan Swindells, MBBS, on November 16, 2021.

TB will also be addressed at the CROI 2022 symposium, TUBERCULOSIS: NEW STRATEGIES IN THE TREATMENT OF THE WORLD’S OLDEST EPIDEMIC, on Wednesday, February 23, 2022. Content from CROI 2022 will be available to the public 1 month following the conference.

How does the risk of myocardial infarction (MI) for people with HIV compare with the risk for people without HIV?

Responses
A. The risk is the same 9% (54)
B. 10% to 20% lower for people with HIV
4% (27)
C. 20% to 100% higher for people with HIV
78% (466)
D. 200% to 400% higher for people with HIV 9% (54)

Correct answer is C. People with HIV have approximately 1.5-fold (150%) higher risk for MI than people without HIV. There is some variation in studies, depending on the cohorts and control populations, but the vast majority have demonstrated people with HIV have anywhere from 20% to 100% higher MI risk than people without HIV.

For more information, click the Read Article button below to read the article, “HIV and Cardiovascular Disease: From Insights to Interventions,” authored by Matthew J. Feinstein, MD, MSc, which was published in the October/November 2021, Volume 29, Issue 4 of Topics in Antiviral Medicine.

A 32-year-old cisgender woman with HIV on effective antiretroviral therapy who you have been following for 10 years is diagnosed with primary syphilis. Her rapid plasma reagin (RPR) titer is 1:256. She is treated with 2.4 μL of intramuscular benzathine penicillin. At 3 months, her titer is 1:256; at 6 months, her titer is 1:128; and at 9 months (currently), her titer is 1:128. Her examination is normal, and she denies any re-exposures. What is the most appropriate management approach at this time?

 

Responses
A. Benzathine penicillin G 2.4 μL intramuscular 1 time 7% (31)
B. Benzathine penicillin G 2.4 μL intramuscular 3 times 27% (122)
C. Continue to observe 43% (197)
D. Cerebrospinal fluid examination 23% (109)

Correct answer is C. The current Sexually Transmitted Infection (STI) Treatment Guidelines recommend waiting a full 12 months for persons with early syphilis and a full 24 months for persons with late latent syphilis before anticipating what to do with serological titers following therapy. In this situation, the patient was treated 9 months ago for early syphilis. Her titer has declined 2-fold (or 1 dilution) so far. She denies any re-exposures. The most appropriate approach is to continue to observe for at least a full 12 months. Interestingly, titers in persons with HIV may decline more slowly. The STI Treatment Guidelines acknowledge this and suggest that clinicians may wish to wait for a full 24 months for people with HIV treated for early syphilis.

“Challenging Diagnostic and Management Questions in Syphilis: A Case-Based Approach,” “Leveraging Administrative and Governmental Policies to Improve PrEP Utilization,” presented by Khalil G. Ghanem, MD, PhD, on November 2, 2021.

In the United States, the rate of people with HIV who are uninsured compared with the general adult population is approximately which of the following?

 

Responses
A. The same 24% (59)
B. Higher
54% (133)
C. Lower 22% (55)

Correct answer is A. Since the Patient Protection and Affordable Care Act (PPACA) was enacted, the rate of people with HIV who are uninsured is roughly equal to the rest of the adult population.

To learn more about the relationship between policy and access to care, click the below link to watch the webinar, “Leveraging Administrative and Governmental Policies to Improve PrEP Utilization,” presented by Jeffrey Crowley, MPH, at the November 19, 2021, virtual course, PrEParing for PrEP: From Policy to Implementation.

Which statement regarding interactions with contemporary antiretrovirals is FALSE?

Responses
A. Oral absorption of integrase strand transfer inhibitors is significantly reduced by polyvalent cations such as aluminum, calcium, magnesium, iron, and zinc that frailty in one clinic is assessed the same in another clinic 13% (16)
B. Potent enzyme and transporter inducers reduce cabotegravir/rilpivirine exposures when cabotegravir/rilpivirine is given orally, but not when given intramuscularly
46% (58)
C. Several antiretroviral drugs inhibit membrane transporters resulting in higher exposures of transporter substrates 28% (36)
D. Iatrogenic Cushing’s syndrome may occur when inhaled, intranasal, intra-articular, or ocular corticosteroids are used in people with HIV on antiretroviral regimens that contain ritonavir or cobicistat 13% (17)

Correct answer is B. Potent enzyme and transporter inducers reduce cabotegravir/rilpivirine exposures both when given orally and when administered intramuscularly.

For more information on how to manage drug-drug interactions and polypharmacy in your clinic, follow the link below to view the presentation, “Managing Polypharmacy and Drug-Drug Interactions,” presented by Jennifer J. Kiser, PharmD, PhD, at the virtual 2021 Ryan White HIV/AIDS Program CLINICAL CONFERENCE.

Which of the following statements is true regarding assessments for frailty or functional limitations?

Responses
A. The Frailty Phenotype and Index have clear-cut points and components that ensure that frailty in one clinic is assessed the same in another clinic 23% (36)
B. The Short Physical Performance Battery can be collected retrospectively
7% (11)
C. The Frailty Phenotype provides a wide range of scores 16% (26)
D. The Frailty Index can be easily determined using just a few variables 15% (24)
E. The Clinical Frailty Scale requires less than a minute to complete 39% (61)

Correct answer is E. The Frailty Phenotype and Index have different-cut points or components depending on the cohort; the Short Physical Performance Battery must be collected in real-time, and the Frailty Phenotype scores range from 0 to 5. Assessments such as the 400-meter walk can take less than 240 seconds to more than 900 seconds to administer. A Frailty Index typically includes at least 50 variables. The Clinical Frailty Scale is a quick “eye ball” test of frailty that is completed in less than a minute.

For more information on how to conduct these assessments in your clinic, follow the link below to view the presentation, “Assessing Frailty in Older People With HIV,” presented by Kristine M. Erlandson, MD, MS, at the June 25, 2021, virtual course, Aging and HIV: Issues, Screening, and Management in Individuals with HIV as They Age.

The recombinant herpes zoster vaccine is recommended for which group of people with HIV?

Responses
A. All people with HIV age 50 years and older 43% (150)
B. All people with HIV age 18 years and older 26% (89)
C. People with HIV and a CD4+ count greater than 200 cells/μL 19% (65)
D. People with HIV who have not had a prior episode of herpes zoster 4% (14)
E. None of the above 8% (27)

Correct answer is B. Herpes zoster is a common infection in people with HIV, and the infection may be severe and recurrent. The Advisory Committee on Immunization Practices (ACIP) and the US Department of Health and Human Services (DHHS) Opportunistic Infection Guidelines have recently been updated to recommend the recombinant herpes zoster vaccine in all adults with HIV infection.

For more information, click the Read Article button below to read the article, “Primary Care Concerns for the Aging Population With HIV”, authored by Steven C. Johnson, MD, which was published in the October/November 2021, Volume 29, Issue 4 of Topics in Antiviral Medicine.

Responses
A. Virologic suppression was not achieved in treatment-experienced patients with low CD4+ count (<200 cells/μL) when they were switched to dolutegravir (DTG) vs darunavir/ritonavir (DRV/r) each with tenofovir/emtricitabine (TDF/FTC) or zidovudine (ZDV)/3TC 10% (40)
B. After NNRTI therapy failure, DTG or DRV boosted with ritonavir (RTV) plus TDF/FTC would be efficacious, regardless of the amount of nRTI resistance 57% (222)
C. Virologic suppression was not achieved in treatment-experienced patients with high baseline viremia (>100,000 copies/mL) when switched to DTG vs DRV/r each with TDF/FTC or ZDV/3TC 13% (51)
D. There should be a minimum of 1 predicted active nRTI for her to achieve virologic suppression on DTG plus TDF/FTC 20% (77)

Correct answer is B. The NADIA trial showed that after NNRTI therapy failure, DTG or DRV/r plus TDF/FTC were efficacious, regardless of the amount of nRTI resistance. Baseline low CD4+ cell count or high viral load were not significant predictors of failure after switching. This study complements the findings of previous trials in treatment-experienced patients such as SECOND LINE (Ritonavir-Boosted Lopinavir Plus Nucleoside or Nucleotide Reverse Transcriptase Inhibitors Versus Ritonavir-Boosted Lopinavir Plus Raltegravir for Treatment of HIV-1 Infection in Adults With Virological Failure of a Standard First-Line ART Regimen) and EARNEST (Nucleoside Reverse-Transcriptase Inhibitor Cross-Resistance and Outcomes from Second-Line Antiretroviral Therapy in the Public Health Approach), which showed that first-line NNRTI failures could be treated with regimens containing an older protease inhibitor (lopinavir/ritonavir [LPV/r]) with 2 or 3 nRTIs or LPV/r plus raltegravir (RAL).

For more information, click the Watch On-Demand Recording button below to view the presentation, “In Case You Missed It: Updates From Recent Publications and Meetings”, presented by Roger J. Bedimo, MD, MS, at the November 5, 2021, virtual course, HIV, COVID-19, and Sexually Transmitted Infections: Update and Implications for Practice.

Responses
A. Start postexposure prophylaxis (PEP) 11% (60)
B. Check HIV viral load and start PrEP today 49% (261)
C. Check HIV viral load and delay starting PrEP 37% (197)
D. Start 3-drug antiretroviral treatment twice daily 3% (16)

Correct answer is B. This patient has been off PrEP for several months, and he reports ongoing potential HIV exposures. He has some symptoms concerning for possible acute HIV (chills, subjective fevers, sore throat), but he has a cough and several sick contacts, which is suggestive of a respiratory infection. The rapid HIV test has a longer window period, so he would be false negative in the setting of acute HIV infection. It would be important to evaluate for acute HIV, and restarting PrEP would be reasonable given the symptoms are most consistent with an upper respiratory infection.

For more information, click the Watch Webcast button below to view the presentation, “Assessing and Monitoring Persons Interested in Starting and Continuing PrEP”, presented by Hyman Scott, MD, MPH, at the November 19, 2021, virtual course, PrEParing for PrEP: From Policy to Implementation.

Responses
A. Raltegravir 1200 mg daily 16% (90)
B. Bictegravir 50 mg daily 14% (83)
C. Lopinavir 400 mg/ritonavir 100 mg twice daily 10% (58)
D. Efavirenz 600 mg daily 60% (342)

Correct answer is D. Standard TB treatment includes rifampin, so multiple drug-drug interactions are possible. Raltegravir should be dosed at 800 mg twice daily; single daily dosing results in unacceptable decreases in concentration. Bictegravir and lopinavir/ritonavir should not be coadministered because of large decreases in antiretroviral concentrations. In contrast, rifampin only leads to modest reduction in efavirenz concentrations at most, so no dose adjustment is needed.

For more information, click the Watch Webinar button below to view the presentation, Update on Tuberculosis/HIV Coinfection: Across the Spectrum From Latent Infection Through Drug-Susceptible and Drug-Resistant Disease, presented by Susan Swindells, MBBS, on November 16, 2021.

Responses
A. A 40-year-old smoker with a 15-pack-year history 19% (148)
B. A 60-year-old with a 40-pack-year history and no other pulmonary issues 40% (309)
C. A 55-year-old with a 35-pack-year history and complaints of chronic cough with occasional hemoptysis 35% (273)
D. A 70-year-old with a 30-pack-year history who quit smoking 20 years prior 6% (50)

Correct answer is B. The USPSTF screening recommendations are for people 50- to 80-years-old with a history of at least 20-pack-years, who are current smokers or have quit within the past 15 years, and currently have no signs of lung cancer.

For more information, click the Watch On-Demand Session button below to view the presentation, “Non-AIDS Cancers,” presented by Timothy J. Wilkin, MD, on October 5, 2021, for the Ryan White HIV/AIDS Program CLINICAL CONFERENCE.

Responses
A. The drug inhibits HIV replication via 2 different mechanisms making resistance less likely 57% (369)
B. The drug obtains high levels in plasma that are maintained over time 12% (81)
C. The drug has a long intracellular half-life 21% (135)
D. None of the above 10% (65)

Correct answer is A. Islatravir has a dual mechanism of action, inhibiting translocation and chain termination. These properties are expected make it more difficult for resistant viruses to emerge with this antiretroviral drug.

For more information, click the Watch On-Demand Session button below to view the presentation, “New and Investigational ART Drugs and Strategies,” presented by Judith S. Currier, MD, on October 3, 2021, for the Ryan White HIV/AIDS Program CLINICAL CONFERENCE.

Responses
A. Oral absorption of integrase strand transfer inhibitors is significantly reduced by polyvalent cations such as aluminum, calcium, magnesium, iron, and zinc 13% (131)
B. Potent enzyme and transporter inducers reduce cabotegravir/rilpivirine exposures when cabotegravir/rilpivirine is given orally, but not when given intramuscularly 43% (417)
C. Several antiretroviral drugs inhibit membrane transporters resulting in higher exposures of transporter substrates 25% (243)
D. Iatrogenic Cushing’s syndrome may occur when inhaled, intra-nasal, intraarticular, or ocular corticosteroids are used in people with HIV on antiretroviral regimens that contain ritonavir or cobicistat 19% (182)

Correct answer is B. Potent enzyme and transporter inducers reduce cabotegravir/rilpivirine exposures both when given orally and when administered intramuscularly.

For more information, click the Watch On-Demand Session button below to view the presentation, “Managing Polypharmacy and Drug-Drug Interactions,” presented by Jennifer J. Kiser, PharmD, PhD, on October 4, 2021, for the Ryan White HIV/AIDS Program CLINICAL CONFERENCE.

Responses
A. Trichomonas can cause urethritis in men who have sex with other men, likely acquired through receptive anal intercourse 16% (107)
B. N. meningitidis has a similar colony morphology appearance on culture and cannot be distinguished from N. gonorrhoeae on Gram stain 27% (178)
C. Herpes simplex virus (HSV)-2 is a more common etiology of urethritis than HSV-1 6% (39)
D. M. genitalium urethritis has been associated with chronic complications among men, including epididymitis, prostatitis, or infertility 51% (339)

Correct answer is B. N. meningitidis has a similar colony morphology appearance on culture and cannot be distinguished from N. gonorrhoeae on Gram stain.

T. vaginalis can cause urethritis among heterosexual men. However, the prevalence varies substantially by US geographic region, age, sexual behavior, and within specific populations.

N. meningitidis has similar colony morphology appearance on culture and cannot be distinguished from N. gonorrhoeae on Gram stain. Identification of N. meningitidis as the etiologic agent with presumed gonococcal urethritis on the basis of Gram stain but negative nucleic acid amplification testing (NAAT) for gonorrhea requires a confirmation by culture. Meningococcal urethritis is treated with the same antimicrobial regimens as gonococcal urethritis.

Nongonococcal urethritis (NGU) can be caused by herpes simplex virus, Epstein-Barr virus, or adenovirus acquired by oral-penile contact. Herpes simplex virus (HSV)-1 infections are usually the most common and may be associated with meatitis or genital ulcerations.

Associations between urethritis and insertive anal and oral exposure among heterosexual men with urethritis have been reported with Leptotrichia or Sneathia species. These studies increase concern for possible undetected infectious rectal or vaginal pathogens, or alternatively, a transient reactive dysbiosis after exposure to a new microbiome or even a noninfectious reactive etiology.

Data are insufficient to implicate M. genitalium infection with chronic complications among men, including epididymitis, prostatitis, or infertility.

For more information, click the Watch Webinar button below to view the presentation, “Exploration of the Latest Update to the Sexually Transmitted Infection (STI) Treatment Guidelines,” presented by Kimberly A. Workowski, MD, on October 19, 2021.

Responses
A. Inflammation 7% (90)
B. Metabolism of antiretroviral drugs 29% (354)
C. C. difficile infection 59% (717)
D. Epithelial barrier damage 5% (60)

Correct answer is C. Inflammation, metabolism of antiretroviral drugs, and epithelial barrier damage have been shown to increase HIV transmission. Although C. difficile infection has been associated with and has been shown in some studies to increase HIV pathogenesis, it has never been shown to increase HIV transmission.

For more information, click the Watch Webinar button below to view the webinar, “The Role of the Microbiome in Modulation of Inflammation and Impact on Disease in HIV and COVID-19,” presented by Nichole Klatt, PhD, on July 27, 2021.

Responses
A. Increased weight 28% (340)
B. Lower CD4+ count 34% (408)
C. Mood disturbance 4% (47)
D. HIV disease progression 19% (222)
E. Increased cardiovascular mortality 15%(176)

Correct answer is A. There are studies that suggest poor sleep in people with HIV is associated with a lower CD4+ count, HIV disease progression, increased cardiovascular mortality, and mood disturbance. However, it is not evident that increased weight is associated with poor sleep in people with HIV.

To learn more about sleep disorders in people with HIV, click the Watch Webinar button below to view the webinar, “Sleep Disorders and Sleep Assessment in People with HIV,” presented by Ana C. Krieger, MD, MPH, on September 21, 2021.

Responses
A. Azithromycin 1 g orally once 11% (66)
B. Azithromycin 1 g orally once a week for 3 consecutive weeks 4% (22)
C. Ceftriaxone 500 mg intramuscularly once 5% (32)
D. Doxycycline 100 mg orally twice daily for 1 week 28% (171)
E. Doxycycline 100 mg orally twice daily for 3 weeks 52%(315)

Correct answer is E. This patient has Chlamydia trachomatis proctitis, which may be caused by the D-K serovars or the lymphogranuloma venereum (LGV) serovars. There is no way to know because additional testing to determine the serovar is not available. The best antibiotic to treat chlamydia proctitis is doxycycline. The treatment duration depends on symptoms when LGV testing is not available. For mild symptoms, 1 week of doxycycline is sufficient. For severe symptoms, 3 weeks is necessary. The patient has severe symptoms of proctitis, and she would need a 3-week course of therapy.

The 2021 CDC STI screening recommendations can be read on the CDC website.

Dr Khalil G. Ghanem will give an in-depth update on STIs and HIV in a presentation titled, “Diagnosing and Managing Sexually Transmitted Infections,” which will be available for on-demand viewing approximately 2 weeks following the virtual Ryan White HIV/AIDS CLINICAL CONFERENCE.

Which statement is true regarding screening or treatment of hyperlipidemia in people with HIV?

Responses
A. A CD4+ count of less than 200 cells/µL is as strong a risk factor for cardiovascular disease as hypertension 10% (141)
B. All people with HIV should be on a statin, regardless of low-density lipoprotein (LDL) cholesterol level 4% (64)
C. Lovastatin and simvastatin are preferred statins with protease inhibitors 7% (101)
D. Prolonged HIV viremia increases cardiovascular risk, requiring adjustment of risk calculator score 79% (1152)

Correct answer is D. Prolonged HIV viremia is a potential HIV-related cardiovascular disease (CVD) risk-enhancing factor and is a consideration for increasing a patient’s risk calculator score.

To learn more about the latest recommendations for preventing and managing CVD in people with HIV, as well as other important updates on primary care for people with HIV, click the Watch Webinar button below to view the webinar, “HIV 101: Fundamentals of HIV Medicine, Initiation of Antiretroviral Therapy, and Primary Care for People With HIV,” which was presented by Melanie A. Thompson, MD, on September 28, 2021.

For an in-depth update on CVD and HIV, click the Watch Webcast to view “Cardiovascular Disease in HIV: Moving From Insights to Interventions,” presented by Matthew J. Feinstein, MD, MSc, on June 25, 2021.

Approximately what proportion of people with HIV experience sleep disturbances compared with the general public?

Responses
A. About half as many 5% (57)
B. About the same 9% (101)
C. About twice as many 72% (779)
D. About 3 times as many 14% (151)

Correct answer is C. Approximately 70% of people with HIV experience some type of sleep disturbance impacting their overall health. This is more than twice the rate of the general public, which is approximately 30%. Sleep problems in people with HIV are associated with lower treatment adherence, lower quality of life, impaired metabolic control, and depression.

For more information, click the Watch Webinar button below to watch the webinar, “Sleep Disorders and Sleep Assessment in People with HIV,” presented by Ana C. Krieger, MD, MPH, on September 21, 2021.

Which HIV preexposure prophylaxis (PrEP) regimen or formulation is NOT under clinical investigation?

 

Responses
A. Daily oral tenofovir alafenamide/emtricitabine (TAF/FTC) 30% (203)
B. Weekly tenofovir disoproxil fumarate/emtricitabine (TDF/FTC) patch 36% (246)
C. Monthly oral islatravir 14% (95)
D. Injectable cabotegravir administered every 2 months 7% (48)
E. Subcutaneous lenacapavir administered every 6 months 13% (90)

Correct answer is B. There is no antiretroviral patch for HIV prevention currently in clinical trials, but several are in pre-clinical development. All of the other PrEP regimens and formulations listed have reached clinical trials.

For more information, click the Watch Webcast button below to watch Rapid Review of New Scientific and Clinical Data Presented at the International AIDS Society (IAS) 2021 Conference, presented by Roy M. Gulick, MD, MPH, on September 7, 2021.

Regarding screening for cognitive impairment in older patients with HIV, which of the following recommendations is shared across commonly used guidelines?

Responses
A. All patients with HIV should have a baseline cognitive screening and repeat screening annually 29% (281)
B. No cognitive screening is recommended 3% (29)
C. There is substantial controversy across guidelines in terms of regular cognitive screening 33% (321)
D. The United States Preventative Service Task Force (USPSTF) recommends cognitive screening for all patients over the age of 60 years regardless of HIV status
35% (342)

Correct answer is C. There is substantial inconsistency across major guidelines, including that of the International Antiviral Society-USA (IAS–USA), the United States Department of Health and Human Services (DHHS), the European AIDS Clinical Society, and the Infections Disease Society of America (IDSA). Recommendations from some groups include frequent screening and others suggest screening in symptomatic patients. Although the USPSTF does not offer recommendations specific to people with HIV, their general guidance on screening for cognitive impairment in the general population states that “the evidence is insufficient to balance the benefits and harms.”

For more information, click the Watch Webcast button below to watch “Neurocognition and the Aging Brain,” presented by Victor G. Valcour, MD, PHD, at June 25, 2021, virtual course, Aging and HIV: Issues, Screening, and Management in Individuals with HIV as They Age.

A 25-year-old transgender woman has just been diagnosed with anal chlamydia in your clinic. You advise her to consider preexposure prophylaxis (PrEP) with daily tenofovir disoproxil fumarate and emtricitabine (TDF/FTC) to reduce her risk for HIV acquisition. She takes oral spironolactone 100 mg twice daily and oral estradiol 2 mg twice daily as part of her gender-affirming hormone regimen. Baseline test results are HIV antigen/antibody negative, hepatitis B surface antigen (HBsAg) negative, and an estimated glomerular filtration rate (eGFR) of 90. She is concerned about the possibility of drug-drug interactions between her hormones and PrEP. Which statement is true regarding the use of PrEP among transgender women?

Responses
A. TDF/FTC has been demonstrated in clinical trials to be as effective in preventing HIV acquisition among transgender women as in cisgender men 53% (344)
B. TDF/FTC is associated with a 10 to 15% reduction in estradiol levels among transgender women receiving gender-affirming hormones 14% (89)
C. Feminizing hormones significantly reduce levels of TDF/FTC and therefore PrEP should be avoided in this population 3% (16)
D. Stricter adherence to daily TDF/FTC in transgender women may be needed to prevent HIV infection 30% (197)

Correct answer is D. Daily oral TDF/FTC is effective in preventing HIV in transgender women when taken as prescribed. There are no known drug-drug interactions between TDF/FTC and gender-affirming hormones, nor are there any known contraindications to concomitant use of PrEP with gender-affirming hormone therapy. There are indications that TDF levels are lower in the presence of estrogen, but the clinical significance is not known. Transgender women on hormone therapy may need to have better adherence to TDF/FTC to be sufficiently protected.

For more information, click the Watch Webinar button below to watch Management and Prevention of HIV Infection Among Transgender Adults, presented by Asa E. Radix, MD, PhD, MPH, on August 18, 2020.

Which of the following is a potential toxic effect of bacteriophage therapy?

Responses
A. Induction of septic shock by accelerated lysis of bacterial pathogens
27% (124)
B. Induction of bacteriophage-specific adaptive immune responses
21% (99)
C. Urine NAATs for gonorrhea and chlamydia, and pharyngeal NAAT for gonorrhea only 17% (77)
D. Inadvertent administration of endotoxin or other impurities in bacteriophage preparations 35% (159)

Correct answer is D. Toxic effects of bacteriophage therapy could theoretically include inadvertent administration of endotoxin or other impurities in bacteriophage preparations. However, very sensitive endotoxin assays are now available and if used during the preparation of the phage for administration, prevent this from being an issue.

To learn more about the potential clinical niches for bacteriophage therapy, click the Watch Webinar button below to watch Bacteriophage Therapy: The Enemy of My Enemy is My Friend, presented by Robert T. Schooley, MD, on August 24, 2021.

A 38-year-old man with HIV has been on antiretroviral therapy for the past 2 years and has had a viral load of less than 200 copies/mL for the last 18 months. He is in a relationship but occasionally has sexual encounters outside of his primary relationship with other men, using condoms for anal sex but not oral. He is asymptomatic of any problems suggestive of sexually transmitted infections (STIs). The appropriate recommendation for STI testing for this individual, based on the United States Department of Health and Human Services (DHHS) and Centers for Disease Control and Prevention (CDC) guidelines, would be:

 

Responses
A. No testing is needed since he has an undetectable HIV viral load and is asymptomatic 2% (18)
B. Urine nucleic acid amplification tests (NAATs) for gonorrhea and chlamydia only 2% (19)
C. Urine NAATs for gonorrhea and chlamydia, and pharyngeal NAAT for gonorrhea only 5% (33)
D. Urine NAATs for gonorrhea and chlamydia, rectal NAATs for gonorrhea and chlamydia, and pharyngeal NAAT for gonorrhea only 8% (54)
E. Urine NAATs for gonorrhea and chlamydia, rectal NAATs for gonorrhea and chlamydia, pharyngeal NAAT for gonorrhea, and syphilis serology testing 83% (592)

Correct answer is E. The updated CDC STI treatment guidelines recommend regular (at least once annually, or 3 to 6 times if at greater risk) testing for gonorrhea, chlamydia, and syphilis for those at higher risk, including people with HIV and men who have sex with men (MSM). For gonorrhea, it is recommended that sites of contact for MSM (urine, rectal, and pharyngeal) be tested regardless of condom use. For chlamydia, it is recommended that sites of contact for MSM (urine and rectal) be tested regardless of condom use. For syphilis, it is recommended that those who are asymptomatic and at higher risk be tested at least once annually.

The 2021 CDC STI screening recommendations can be read on the CDC website.

Which of the following statements is true regarding the HIV reservoir in elite controllers?

Responses
A. Their HIV reservoir is extremely large 6% (37)
B. Integrated proviruses from elite controllers are often transcriptionally active 28% (177)
C. Most intact proviruses accumulate at non-genic and satellite DNA 45% (283)
D. Immune pressure has likely no impact on the reservoir in elite controllers 21% (138)

Correct answer is C. Clonally-expanded intact proviral genomes in exceptional elite controllers accumulate at satellite DNA in centromeric regions and KRAB-ZNF genes on chromosome 19. Elite controllers usually carry a very small frequency of infected cells, most proviruses in elite controllers are integrated in regions of the chromatin that repress gene transcription, and the reservoir of elite controllers is under the pressure of a potent cytotoxic T lymphocyte (CTL) response.

Click the Watch Webcast button to view “Elite Controllers: A Model for a Functional Cure of HIV-1 Infection,” presented by Xu Yu, MD, at vCROI 2021 on March 8, 2021.

Most of the individuals who responded that they were going to “wait and see” when asked if they were planning to get a COVID-19 vaccine in January 2021 have now been vaccinated.

 

Responses
A. True 50% (616)
B. False 50% (607)

Correct answer is A. In January 2021, the Kiser Family Foundation COVID-19 Vaccine Monitor asked individuals if they were going to get a COVID-19 vaccine once they were eligible. In a new poll published on July 13, 2021, of the individuals that responded that they were going to “wait and see,” 54% have now reported that they are vaccinated.

Click the Watch Webinar button to view COVID-19 Vaccine Hesitancy, Crucial Conversations, and Effective Messaging for Patients and Healthcare Teams, presented by Marie T. Brown, MD, and moderated by Constance A. Benson, MD, IAS-USA Webinar, on August 3, 2021.

As highlighted in the NA-ACCORD (North American AIDS Cohort Collaboration on Research and Design) study, which of the following was associated with an increased risk for hepatocellular carcinoma (HCC) in people with HIV/HBV coinfection?

 

Responses
A. Detectable HIV viral load 17% (171)
B. Low CD4+ cell percentage 14% (146)
C. Hepatitis B virus (HBV) DNA level above 200 IU/mL 50% (505)
D. Hepatitis delta infection 12% (126)
E. Race 7% (71)

Correct answer is C. Even a low level of HBV viral replication was associated with an increased risk of HCC. Undetectable HBV for more than a year was associated with lower HCC rate, compared with detectable HBV or undetectable for less than a year. This highlights the importance of monitoring for HBV suppression in HIV/HBV coinfection as the data suggests sustained suppression of HBV replication may be key to reducing HCC risk, rather than simply getting HBV below a certain threshold.

Click the Read Article button to read “CROI 2021: Viral Hepatitis and Other Forms of Liver Injury Impacting People with HIV,” authored by Anne F. Luetkemeyer, MD, and David L. Wyles, MD, in IAS-USA Topics in Antiviral Medicine, Volume 29, Issue 3.

Which of the following statements is true?

 

Responses
A. Anal cancer screening has been shown to reduce the rates of anal cancer in people with HIV (PWH) 42% (269)
B. Rates of anal cancer in PWH approximate or exceed the rates of lung, colon, and cervical cancer in the general population 47% (302)
C. Among women with HIV, only those with certain risk factors should be screened for anal cancer precursors or anal high grade squamous intraepithelial lesions (HSIL) 11% (72)

Correct answer is B. Among men who have sex with men with HIV, the rates of anal cancer match or exceed those of colon, lung, and possibly prostate cancer. Among women with HIV, the rate of anal cancer approximates historic cervical cancer rates prior to widespread cervical cancer screening.

Click the Watch Webinar button to view Prevention of HPV-Related Anal Cancer in Persons With HIV: How Should We Screen and Will Vaccines Make a Difference?, presented by Grant Ellsworth, MD, MS, at the July 13, 2021, IAS-USA webinar.

Which of the following is a valid reason to check for SARS-CoV-2 spike antibody levels in a vaccinated patient?

Responses
A. To find out if they are protected 18% (177)
B. To find out if a booster shot is needed 16% (150)
C. For research purposes only 63% (609)
D. Because your patient wants to know 3% (26)

Correct answer is C. More research is needed to determine if spike binding antibodies can be useful for determining if a patient is immune-protected or needs a booster. This is currently unknown. Both the United States Center for Disease Control and Prevention (CDC) and United States Federal Drug Administration (FDA) do not recommend that post-vaccination antibody levels be routinely checked, since we do not know how to interpret that information, how to act on it, and the commercial antibody assays are not authorized for that use.

Commercial spike antibody assays are not authorized (nor are they proven to have predictive value) by the FDA for determining if a person has immune-protection against SARS-CoV-2 and variants, and may or may not need a booster shot. The authorized COVID-19 vaccines induce functional antibody (Ab) and cellular responses. Commercial assays are qualitative or semi-quantitative measures of binding antibodies only, not measures of functional antibodies (eg, neutralizing Ab, antibody-dependent cellular cytotoxicity [ADCC]) and/or cellular immunity.

Healthcare practitioners can educate their patients who want to know their antibody levels and request post-vaccination testing about the above facts; and, where available, refer them to research studies currently investigating the duration of immunity post-vaccination and the utility of booster shots.

Click the Watch Webinar button to view “COVID-19 Vaccines and the Viral Variants: New Aspects of Vaccine Research,” presented by Mark J. Mulligan, MD, at the June 29, 2021, IAS-USA webinar.

As of July 5, 2021, which of the following individuals is eligible for monoclonal antibody treatment under the Emergency Use Authorization (EUA)?

Responses
A. A 66-year-old woman with a positive SARS-CoV-2 polymerase chain reaction (PCR) but no symptoms and a body mass index (BMI) of 30 but no comorbidities 11% (53)
B. A 66-year-old woman with a positive SARS-CoV-2 PCR, a mild cough starting 2 days prior, and no comorbidities, but lives with her husband who has diabetes 14% (69)
C. A 66-year-old man with diabetes who was exposed to his wife who has a positive SARS-CoV-2 PCR but his PCR was negative 9% (43)
D. An 86-year-old man with a positive SARS-CoV-2 PCR, is hospitalized with COVID-19 pneumonia, and has a cough, but is not requiring oxygen 35% (171)
E. An 86-year-old man with no comorbidities and a positive SARS-CoV-2 PCR with mild fatigue starting 2 days prior 31% (153)

Correct answer is E as of the posting of the question on July 5, 2021.The EUA of monoclonal antibodies is for the treatment of COVID-19 in people who are at high risk of disease progression (people who are older and have comorbidities) and not already hospitalized.

Please note, the field of COVID-19 changes rapidly and the half-life of information is short.

Click the Watch Webcast button to view “Preventing Severe COVID-19: The Role of Monoclonal Antibodies and Beyond,” presented by Davey Smith, MD, at the May 20, 2021, virtual course, Unique Issues in HIV Prevention, Treatment, and Care in the COVID-19 World

For asymptomatic individuals with HIV at average risk for colon cancer, what age group does the United States Preventive Services Task Force (USPSTF) recommend routine colon cancer screening?

Responses
A. 50-85 years 9% (62)
B. 45-85 years 13% (88)
C. 50-75 years
18% (122)
D. 45-75 years 39% (263)
E. Beginning at 50 years with no upper age limit 21% (138)

The correct answer is D. In May 2021, the USPSTF broadened the age range that adults should be routinely offered colon cancer screening. For individuals 50 to 75 years old, the recommendation strength is “A,” and for individuals 45 to 49 years old, the recommendation strength is “B.” The USPSTF continues to recommend selective screening for individuals 76 to 85 years old, taking into consideration the patient’s overall health, prior screening history, and preferences, with a recommendation strength of “C.” The HIV Medicine Association (HIVMA)/Infectious Diseases Society of America (IDSA) HIV Primary Care Guidance recommends following the USPSTF recommendations for colon cancer screening for persons with HIV.

Which of the following drugs could cause postural hypotension in an elderly HIV-infected patient?

Responses
A. Dolutegravir 7% (74)
B. Efavirenz 9% (82)
C. Tamsulosin
70% (687)
D. Fexofenadine 5% (46)
E. Sertraline 9% (89)

The correct answer is C. Tamsulosin is an alpha1 antagonist, and can cause postural hypotension. Use caution whenever prescribing alpha antagonists in the elderly, who may have greater risk for cardiovascular instability.

The course presentation covering this topic, along with other presentations that cover some of the key issues unique to management and care of older patients with HIV, will be available for viewing on demand on the IAS–USA website. Those who did not participate in the live activity can earn continuing education credits for viewing the on-demand webcasts.

To learn more about this topic and other issues unique to managing the care of older patients with HIV, watch webinars of June 25, 2021, course, Aging and HIV: Issues, Screening, and Management in Individuals with HIV as They Age.

A 33-year-old man with well-controlled HIV infection was diagnosed with gonococcal infection by his primary HIV practitioner. STI testing had been ordered by the patient’s primary HIV practitioner at a telehealth visit. However, the primary practitioner’s office is not open for in-person visits, and so the patient is referred to your sexual health center for treatment. He brings the results of gonorrhea and chlamydia nucleic acid amplification testing, which show positive oropharyngeal testing for gonorrhea and negative chlamydia oropharyngeal testing. Also negative are rectal and urine gonorrhea and chlamydia testing, as well as syphilis treponemal testing. How do you manage this patient according to the updated Centers for Disease Control and Prevention (CDC) recommendations from December 2020?

Responses
A. Ceftriaxone 250 mg, followed by a test of cure in 7 to 14 days 7% (57)
B. Ceftriaxone 250 mg, no subsequent test of cure but retesting in 3 months to assess for reinfection 8% (69)
C. Ceftriaxone 500 mg, followed by a test of cure in 7 to 14 days 54% (453)
D. Ceftriaxone 500 mg, no subsequent test of cure but retesting in 3 months to assess for reinfection 31% (255)

Correct answer is C. The updated CDC recommendations for treatment of uncomplicated gonococcal infection of the pharynx is ceftriaxone 500 mg, administered as a single intramuscular dose for those weighing less than 150 kg (or 300 lbs). For those weighing more, 1 gm of ceftriaxone should be administered. If chlamydia infection has not been excluded, treatment should also include doxycycline 100 mg twice daily for 7 days (unless the patient is pregnant, and then 1 gm of azithromycin should be administered for chlamydia treatment). A test of cure, using culture or nucleic acid amplification testing, is recommended at 7 to 14 days after treatment for pharyngeal infection. Test-of-cure is not thought to be necessary after treatment for uncomplicated urogenital or rectal gonorrhea when a recommended regimen is used.

Click the Watch Webcast button to view “Management and Prevention of Sexually Transmitted Infections,” presented by Meredith Clement, MD, at the April 30, 2021, virtual course, Annual Update on HIV Management: State-of-the-Art Updates on HIV, STIs, and COVID-19.

A 52-year-old man with HIV is seen in clinic for a routine visit. He received his second dose of COVID-19 vaccine (Moderna product) 4 days ago with only a mild reaction and currently is asymptomatic. He is scheduled to get a dose of pneumococcal polysaccharide vaccine (PPSV-23) at this visit. What would you recommend regarding administering the PPSV-23 vaccine for this man?

Responses
A. He must wait at least 14 days after the most recent COVID-19 vaccine dose 40% (419)
B. He must wait at least 6 weeks after the most recent COVID-19 vaccine dose 6% (66)
C. He must wait at least 12 weeks after the most recent COVID-19 vaccine dose 3% (36)
D. He can receive the PPSV-23 vaccine at this clinic visit
51% (540)

Correct answer is D. Prior to May 14, 2021, the Centers for Disease Control and Prevention (CDC) recommended that no vaccines be coadministered within 14 days (before or after) any dose of the COVID-19 vaccine. These recommendations changed on May 14, 2021, and the CDC guidance now provides the following guidance (verbatim) regarding coadministering vaccines with COVID-19 vaccines:

  • COVID-19 vaccines and other vaccines may now be administered without regard to timing.
  • COVID-19 vaccines and other vaccines on the same day, as well as coadministration within 14 days.
  • If multiple vaccines are administered at a single visit, administer each injection in a different injection site.

This guidance does not mean that a practitioner cannot elect to defer a vaccine, which may be prudent in some situations, such as coadministering a dose of non-COVID-19 vaccine with the COVID-19 vaccine on the same day if the non-COVID-19 vaccine has a potent conjugate or adjuvant (eg, recombinant zoster vaccine or CPG-HepB) that may enhance the reactogenicity to the COVID-19 vaccine (or create highly uncomfortable/problematic combined postvaccine symptoms). Also, some practitioners may elect to delay the non-COVID-19 vaccine if it can easily be delayed and there is no eminent threat from that pathogen. In general, given the current state of the COVID-19 epidemic, the COVID-19 vaccine should always have priority for administration over a non-COVID-19 vaccine.

Click the Watch Webcast button to view “Vaccine Prevention for Individuals With HIV in the Era of COVID-19,” presented by David H. Spach, MD, at the May 20, 2021, virtual course, Unique Issues in HIV Prevention, Treatment, and Care in the COVID-19 World.